Download as pdf or txt
Download as pdf or txt
You are on page 1of 198

1

6
Mathematics
Learner’s Activity Material

By:
Maria Carmela S. Dizon
Maria Palma V. Lopez
Analie J. Maquirang
Nimfa B. Tapnio

This module is written in support of the K to 12 Basic Education Curriculum. This


is to assure that the learners achieve the expected learning competencies for the
Grade Level.

Department of Education
Republic of the Philippines

2
INTRODUCTION

This module is written for the learners to enjoy this new learning
experience that support the K to 12 Basic Education Curriculum. This
is to guarantee that the students achieve the expected standard
learning competencies for the Grade Six Level.

This material provides understanding and appreciation of skills


involving Fractions, Decimals, Ratio and Proportion, Percent, Integers,
Geometry, Patterns and Algebra, Measurements, and Statistics and
Probability. Each lesson consists of activities and exercises to help the
learners to understand and to master the mathematical concepts and
principles. It also encourages the learners to use these concepts and
principles in real – life situations.

With the activities provided in this module, the writers hope that
it will develop the critical-thinking and problem-solving skills of the
learners by being engaged and challenged to finish all the activities.

3
TABLE OF CONTENTS

CHAPTER 1 FRACTIONS

Lesson 1 Addition of Fractions 7


Lesson 2 Addition of Mixed Fractions 11
Lesson 3 Subtraction of Similar Fractions 15
Lesson 4 Subtraction of Dissimilar Fractions 20
Lesson 5 Multiplication of Fractions 24
Lesson 6 Division of Fractions 29

CHAPTER 2 DECIMALS

Lesson 1 Addition of Decimals 33


Lesson 2 Subtraction of Decimals 37
Lesson 3 Multiplication of Decimals 40
Lesson 4 Division of Decimals 44

CHAPTER 3 RATIO AND PROPORTION

Lesson 1 Concept of Ratio 49


Lesson 2 Concept of Proportion 54
Lesson 3 Types of Proportion 58

CHAPTER 4 PERCENT

Lesson 1 Understanding Percent 62


Lesson 2 Percentage, Rate and Base 67
Lesson 3 Solving Word Problems Involving Percent 70

CHAPTER 5 INTEGERS

Lesson 1 The Set of Integers and the Number Line 76


Lesson 2 Addition and Subtraction of Integers 81
Lesson 3 Multiplication and Division of Integers 87
Lesson 4 Exponents and Exponential Notation 91
Lesson 5 Order of Operation involving integers 95

4
CHAPTER 6 GEOMETRY

Lesson 1 Solid Figures 98


Lesson 2 Nets of Solid Figures 109

CHAPTER 7 PATTERNS AND ALGEBRA

Lesson 1 Writing Rules for Sequences 116


Lesson 2 Algebraic Expressions 125
Lesson 3 Algebraic Equations 137
Lesson 4 Solving Equations 141

CHAPTER 8 MEASUREMENT

Lesson 1 Speed, Distance and Time 148


Lesson 2 Area 153
Lesson 3 Surface Area 157
Lesson 4 Volume 162
Lesson 5 Meter Reading 168

CHAPTER 9 STATISTICS AND PROBABILITY

Lesson 1 Collecting Data 172


Lesson 2 Pie Graphs 178
Lesson 3 Word Problems 183
Lesson 4 Making Simple Predictions 188
Lesson 5 Probability 192

5
6
Lesson Addition of Fractions
1.1
Name:
Level: Section: Date:
Title of the Activity:
Learning Competency: Adds and subtracts simple fractions and mixed numbers
without regroupings
Code: M6NS-la-1.0

In this lesson, one will learn how to add similar and dissimilar
fractions. It will also teach you how to analyze problems involving
addition of fractions.

Objectives:

Add similar fractions


Add dissimilar fractions
Solve word problems involving addition of fractions

Before we proceed in our topic, let us see if you can still remember this
concept.

Find the missing term to make equivalent fractions.


3 6 3 5 35 3
a. 4 = b. = c. = d. =63
12 4 49 9

We already know how to add similar fractions in lower grade. When the
fractions have a same denominator, just add the numerator and write the sum
over the common denominator and express the sum in lowest term.

Example: Find the sum of 3 and 8


10 10
Solution: 3 + 8 = 3+8 = 11 or 1
10 10 10 10 1 10
Now, let us try if you can solve these.

7
4 2 3 7
a. 12 + 12 = c. 15 + 15 =

5 3 6 5
b. 8 + 8 = d. 7 + 7 =

What if the fractions are dissimilar? What should we do to get their sums?

Let us look at the problem situation below


3 4
Jay-Ar ran 5 of a kilometer on Monday and of a kilometer on Thursday. How
7
far did he run in two days?

3 4
Solution: The fractions 5 and 7 are dissimilar fractions. To find the total
3 4
distance he ran, we need to add and . Since they have
5 7
different denominators, we need to change the dissimilar
fractions to similar fractions by getting the Least Common
Denominator (LCD) before adding them. Let us study these
steps.

Step 1: Step 2: Step 3:


Rename the fraction
Find the Add. Write the sum
to equivalent fraction
LCD in the lowest term.
using LCD.

3 3x7 21 3 21
3 = = = 35
5 5x7 35 5
+ 5 20 4 20
4 4 4x5 =
= 35 7 = 35
7 7 7x5

LCD = 35 41 = 1 6
35 35

6
Thus, Jay-Ar jogged 1 kilometers in all.
35

Analyze this example also.

8
5 5x2 10
6 = 6x2 = 12
+
3 3x3 9
= =
4 4x3 12
19 7
LCD = 12 or 1 12
12

Can you solve these?


4 2 5 7
a. + 7 b. + 3 c. + 8 d. + 9
4 3
5 3 4 6
4 6

Let’s Work Together

Find the sum. Then simplify your answers.

1) 3
4 + 1
8

2) 5
9 + 6
9

3) 1
6 + 1
12

4) 1
2 + 3
4 + 3
4

5) 1
2 + 1
3 + 1
6

9
Do on Your Own

Read and analyze the statements then solve mentally.

4 1
1) What is the value of N in 5 + 5 = N?

3 2 1
2) The sum of 5 and 5 added to 5 .

6 3
3) If 12 is added to 12 , what is the answer?

Let’s analyze

Solve each problem.


4
1) Cutting a wood takes hours. Completing the whole
5
3
frame takes additional hours. How long did you work
5
on the frame?
7 4
2. 2) A boy jogged 9 kilometer in the morning and 6 kilometer
in the afternoon. How many kilometers did he run in all?

10
Lesson Addition of Mixed Fractions
I 1.2
Name:
Level: Section: Date:
Title of the Activity:
Learning Competency: Adds and subtracts simple fractions and mixed numbers
without regrouping
Code: M6NS-Ia-1.0

In this lesson, one will learn how to add mixed fractions. It will also
teach you how to analyze problems involving addition of fractions.

Objectives:

Add mixed fractions


Solve word problems involving addition of fractions

In the previous lesson, we discussed on how to add simple fractions, now, in


this lesson you will learn how to add mixed numbers or mixed fractions. Mixed
fractions, or others called it mixed numbers, are kind of fractions composed of
whole number and a proper fraction.

But before we add mixed fractions, can you still change improper fractions to
mixed number? Let us see if you can still do it. Try these exercises.

a. 13 c. 11 = Remember: To change
9 =
2 improper fraction to mixed
number, you need to divide
14 =
b. 8 = D. 22 the numerator by the
4 denominator.

Good! You already know how to change improper fraction to mixed number.
You need this skill in simplifying your answers in solving fractions.

11
Let us go back in addition of mixed fractions.

In adding mixed numbers or mixed fractions, we add the whole numbers then
the fractions. If the fractions are dissimilar change it first to similar fractions.

Let us look at this example.

6 5 6+5 11 2
1 + 4 = 1+4 = 5 = 6
9 9 9 9 9

Since the mixed fractions are similar, we add first the whole numbers. Then
we add the numerators, and copy the denominator. Finally, we simplify the
answer by changing improper fraction to mixed numbers.
Another example is here.
These are dissimilar
4 4x2 8 fractions, change first the
2 = 2 = 2 10
5 5x2 fractions to similar by getting
+ 1 + 5 the LCD. Then proceed in
33 1x5
2 = 3 = 3 10 addition.
2x5
LCD = 10 13 3
5 = 6 10
10

Let’s try these. Find the sum.

3 9 5 3
1 8 9 10 6 6 2 7
+ + 4 + 7 + 5
6 83 33 33
23 10 9 8
8

12
Let’s Work Together

4 2
1. What is the sum of 3 5 and 2 5 ?

2 5
2. 9 12 + 5 8 = N

1 1
3. What is the sum of 7 7 and 6 6 ?

4 12
4. 5 14 + 6 14 = N

1
5. What will you get if 2 2 is added to
1 3
the sum of 3 4
and 44?

Do On Your Own

b. d.
3 8
3 10 9
a. c. 1
+ 1 +
3 63 0
3
3 2 1 43
8 7 4 4
+ 1 +
23 3
8 63
6

13
Let’s analyze

Solve the following problems.


1 1
1. Pamela measured a mixture of 2 2 cups of water and 1 3
cups of milk. How much liquid did she measure then?
3 1
2. She also measured 3 4 cups of flour and 1 cups of sugar.
5
How much mixture does she have?
3 1
3. For a recipe she needs 4
cups of white sugar and 1 3 cups of
brown sugar. How much sugar does she need?
2
4. In one recipe she needs dozen of eggs, and for another
3
4
she needs dozen of eggs. What part of dozen will she
6
need? How many eggs would that be?

14
Lesson Subtraction of Similar Fractions
1.3

Name:
Level: Section: Date:
Title of the Activity:
Learning Competency: Adds and subtracts simple fractions and mixed numbers
without regrouping
Code: M6NS-Ia-1.0

In this lesson, one will learn how to subtract similar fractions without
or with regrouping.

Objectives:

Subtract similar fractions without regrouping


Subtract similar fractions with regrouping

In subtracting similar fractions, subtract the numerators and copy the


denominator then simplify your answer.

Try to study these examples.


10 4 10 – 4 6 1
1. 12 ‒ = = =
12 12 12 2
13 9 13 – 9 4
2. ‒ = =
15 15 15 15
8 3 8‒3 5 1
3. 7 ‒ 4 = 7‒4 = 3 10 = 3
10 10 10 2
6 5 6‒5 1
4. 6 8 ‒ 2 8 = 6‒2 = 4 8
8

15
Now, it’s your turn.
1.
13 9

18 18 =
2.
9 4
‒ =
3. 12 12
9 3
10 13 ‒ 4 =
4. 13
6 2
7 8 ‒ 3 8 =

2 6
What if you have this kind of equation 5 ‒ 3 ? How do you
7 7
solve for this?

Since the numerator of the minuend is less than the numerator of the
subtrahend, we need to regroup the fraction. We need to rename the minuend
to its equivalent fraction.

2 7 2 9
5 7 = 4 + = 4
7 7 7
7 new minuend
5 = 4 7
9 6
The new equation is 4 ‒ 3 . You can now subtract.
7 7

9 6 9‒6 3
4 ‒ 3 = 4‒3 = 1
7 7 7 7

Let us have another example.


3 5
8 ‒ 2 We cannot subtract the numerator. We need to
6 6
rename the minuend by changing it to its
6
8 = 7 equivalent fraction.
6
6 3 9 is the new minuend
7 6 + 6 = 7 6

9 5 9‒5 4 2
7 ‒ 2 = 7‒2 6 = 5 = 5
6 6 6 3

16
Can you try these?

4 8 2 4
a.. 11 ‒ 9 = c. 9 ‒ 7 =
10 10 5 5

b. 1 3 d. 3 5
6 ‒ 3 = 8 ‒ 2 =
4 4 8 8

Since you can answer the previous exercises, this one is easy for you. The
minuend is whole number. Rename it by changing it to a mixed number
before you subtract.

3 4 5
9 ‒ 5 10 ‒ 8 7 ‒ 3 12
7 6

7 3 6 4 12 5
8 ‒ 5 9 ‒ 8 6 12 ‒ 3 12
7 7 6 6
7‒3 6‒4 12 ‒ 5
8‒5 7 9‒8 6 6‒3 12
4 2 1 7
3 1 6 = 1 3 12
7 3

These exercises are made for you.

1. 3 2. 5 3. 4
8 ‒ 4 15 ‒ 9 6 ‒ 1 10
8 9

17
Let’s Work Together

Do as directed.
2 4
1. Take away 7 from 11 6.
6

3
2. What is 4 less than 3 ?
4

5 7
3. If you decrease 8 by 3 9 , what will remain?
9

1
4. Diminish 6 by 4 .
4

5. What is the difference of 7 3/10 and 4 4/10?

Do on Your Own

Find the difference. Reduce answer to lowest term when needed.

19 5
‒ 7 1 1 2
24 24 8 ‒ 3 3 ‒ 1
8 8 3 3

2 4 4 7
6 ‒ 1 9 ‒ 4 9 10 ‒ 5
5 5 12

18
Let’s analyze

Solve the following word problems.


8 6
1. It took Reema 12 of an hour to clean the house and Pamela of
12
an hour to the wash the clothes. How much longer did Reema
work than Pamela?

5
2. Juanito landscaped his garden for 4 6 hours, while Rosita
3
landscaped the same garden for 2 hours. Who works faster? By
6
how many hours?

1 3
3. Corazon bought 5 meters of cloth for a curtain and used only 3
2 4
meters. How many meters were left?

8
4. A fisherman caught 10 kilograms of Tilapia. He sold 5 kilograms
10
and cooked the rest. How many kilograms of Tilapia were cooked?

19
Lesson
1.4 Subtraction of Dissimilar Fractions

Name:
Level: Section: Date:
Title of the Activity:
Learning Competency: Adds and subtract simple fractions and mixed numbers with
regroupings
Code: M6NS-Ia-1.2

In this lesson, one will learn how to subtract dissimilar fractions


without or with regrouping.

Objectives:

Subtract dissimilar fractions without regrouping


Subtract dissimilar fractions with regrouping

Remember that we cannot subtract dissimilar fractions immediately. First, we


need to change it to similar fractions by getting the Least Common
Denominator (LCD) and get the equivalent fractions. Once the fractions are
similar, we can now subtract it.

Look at these examples.


Rename the fraction Subtract. Write the
Find the difference in the
to equivalent fraction
LCD lowest term.
using LCD.

3
3 3x3 9 3 9
4 = = =
4 4x3 12 4 12
‒ 2 ‒ ‒
2 2x2 4 2 4
6 = = =
6 6x2 12 6 12

LCD = 12 5
12

20
Let us have another example.
7 21
7 7x3 21 5 = 5 24
7 5 = 5 = 5 24 8
5 8 8x3 ‒

8 ‒ 3 12
12 2
3 3x4 23 = 3 24
23
3 23
8 = 23
6x4 = 23 24 6
6
9 3
LCD = 2 3 = 3
24 8

Here is another example for you. Opps. We cannot


2 2 2x2 4 subtract the
7 7 = 7 = 7
9 9 9x2 18 numerators.
‒ ‒ Therefore, we need
5 5x3 15
LCD3= 185 33 33 = 33 to rename the
6 6 = 6x3 18
4
minuend 7 18 .

4 15 22 15 7
7 18 ‒ 33 = 6 ‒ 3 = 3
18 18 18 18

18 4 22
7 = 6 18 + 18 = 6 18

Can you solve now? Let us see. Solve the following exercises.
9 3 1
1. 2. 8 3. 7 5
12 4 ‒

‒ 4 2
5 23 13
6 4
8

21
Let’s Work Together

Do as told.
3 1
1. Take away 5 4 from 10 2 .

5 1
2. What will remain if you diminished 8 from 2 3 ?

8 2
3. Subtract 1 20 from 5 10

3 5
4. What is the difference of 6 and 2 6 ?
24

8 3
5. What is 7 less than 20 ?
9 4

Do On Your Own

Find the difference. Reduce answer to lowest term when needed.

1. 2. 3.
10 4 7
7 6 10
15 5
‒ ‒
‒ 3 2 4
3 6 23
6 5

4. 5. 6.
8 6 1
10 11 4
9 9
‒ ‒
‒ 1 43
3 5
6 13
2 2 8

22
Let’s analyze

Solve the following word problems.

1 2
1. A farmer harvested 10 4 sacks of corn. He sold 7 sacks.
10
How many sacks of corn were left?

2 1
2. Marites had 5 3 yards of lace. She needed 9 meters. How
4
much more meter of lace does she need to buy?

3
3. Shanella has 20 4 kilograms of lanzones to sell. She sold
1
15 2½kilograms. Find the weight of lanzones left.

23
Lesson Multiplication of Fractions
1.5

Name:
Level: Section: Date:
Title of the Activity:
Learning Competency: Multiplies simple fraction by another simple fraction or by a
mixed number
Code: M6NS-Ib-4.1

In this lesson, one will learn how to multiply fractions in simple and
mixed forms.

Objectives:

Multiply fractions by another fractions


Multiply fractions by whole numbers
Multiply whole number by mixed numbers

Read the problem below.

3 1
Ricardo painted 5 of the fence. He repainted of
4
painted fence with another color. What part of the fence
had he painted twice?
1 3
To solve this, we need to know what is 4
of 5
.
Remember, the word “of” in Mathematics means
1 3 1 3
multiplication. Therefore of means x 5.
4 5 4

Study the illustration.

3 1 3
of the fence was painted of had been repainted
5 4 5

24
1 3 3
You can count that of 5 is 20 .
4

Therefore, Ricardo painted 3/20 of the fence twice.


1 3
What do you think is the shorter way to solve x ?
4 5

Notice that when we multiply fractions, we simply multiply the numerator by


numerator and then multiply the denominator by denominator.

1 3 1x3 3
x = =
4 5 4x5 20

2 3
What is x
3 4 ?
2 3 2x3 6 1
x = = =
3 4 3x4 12 2

Let us read another problem.

4
Maria answered 5 of the question correctly. There are 20 questions.
How many questions did she get correctly?

4
The number sentence of this problem is x 20. .
5

4
x 20 Rename the whole number 20 as fraction by having a
5
denominator of 1. Remember, we use 1 as the
4 20 denominator of any whole number.
x
5 1
4 x 20 80
= = 16
5x1 5

Maria answered 16 questions correctly.

Here is another problem. Read it carefully then find out what to do.

25
1
Jodi’s mother worked in their sari-sari store for 3 2 hours each day for 4
weeks. How many hours did she spend in their sari-sari store?

1
To solve, we multiply 3 hours by 28 days. We used 28 days because
2
there were 4 weeks given, wherein, in a week there are 7 days.

( 4 weeks x 7 days = 28 days)


1 x 28
3 2 Rename 28 as fraction. Use 1 as denominator.
.
7 28 1
x Rename the mixed number 3 as an improper
2 1 2
fraction.
7 x 28 1 (2x3)+1 7
3 = =
2x1 2 2 2

196
2 = 98

Jodi’s mother worked 98 hours in their store in 4 weeks.


Sometimes you can simplify the multiplication of fractions by using
cancellation method. You can look for the Greatest Common Factor (GCF)
of the numerator and the denominator. This GCF will be used to divide both of
them. Let us try cancellation using the previous examples.
a.
1 1
2 3 1x1 1 The GCF of 3 and 3 is 3. Divide both of
x 4 = = them by 3.
3 1x2 2
1 2 The GCF of 2 and 4 is 2. Divide both of
them by 2

b.
4
4 20 4x4 16 The GCF of 5 and 20 is 5.
x =
1x1 = = 16
5 1 1
1

14
c. 7 28 7x14 98
x =
2 1 1x1 = 1 = 98 The GCF of 2 and 28 is 2.
1

With the use of cancellation method, multiplication of fractions became easier

26
Let’s Work Together

Multiply across and multiply downward to complete the table.

4 5 1 1 2 1
5 6 4 6 1 4 3 2

10 10 8 4 4 2
12 20 14 16 2 6 4 3

Do On Your Own

Find the product.


1. 2. 3.

18 12
x 2 1 1 2
24 15 2 x 1 3 x 1
8 9 3 5

4. 5. 6.
12 9 4
x 9 x 4 9 9
18 21 10 x 20

27
Let’s analyze

Solve the following word problems.


1
5. There are 36 boys in a class. If of them are football team
2
members, how many boys are members of the football team?
How many are not members?

1
6. Each of the 8 boy scouts needs 5 meters of rope for their
2
camping. What is the length of the whole rope needed by
them?

3
7. A cookie recipe needed for 4 of a cup of honey. How much
1
honey is needed to make 2 of a cookie recipe?

5 2
8. On Friday, of the pupils were present. Of these pupils,
6 3
passed their project in Math. What part of the entire class
passed their Math project?

28
Lesson
Division of Fractions
1.6

Name:
Level: Section: Date:
Title of the Activity:
Learning Competency: Divides simple fraction by another simple fraction or by a
mixed fraction
Code: M6NS-Ic-7.1

In this lesson, one will learn how to divide fractions in simple and
mixed forms.

Objectives:

Divide fractions by another fractions


Divide fractions by whole numbers
Divide whole number by mixed numbers

Before we divide, we need to recall the reciprocal. Two fractions are reciprocal
of each other if their product is 1. Like
3 4 3x4 12
x = = = 1
4 3 4x3 12
6 9 6x9 54
x = = = 1
9 6 9x6 54

Now, can you answer these orally. Give the reciprocal of each fraction.

4 13 1 *If it is mixed number, change


12 41 3 2 first to improper fraction
before you get the reciprocal.
6 60
22 90 28 * If it is whole number,
rename it as a fraction by
using 1 as denominator before
you get the reciprocal.

29
I think you are now ready to study this problem.
1
Emma wants to give 2 of papaya to each of
her nearest neighbor. If she has 4 papayas, how
many neighbors can she serve?
1
The problem tells us to divide 4 by . We
2
1
would like to know how many halves ( 2’s) are
there in 4 whole. The number sentence will be

÷ 1
4 = N
2

1
How many halves ( s) are there in 4? The figure above shows that there are
2
1
eight ‘s in 4 or
2
÷ 1
4 = 8
2

1
Do you notice that when we divide 4 by is just like multiplying it by 2?
2
1
What can you say about 2 and 2? Are they reciprocal?
Yes, they are, because when we multiply them, their product is 1.

Here is another example.


3
Analie has a satin cloth of meter long. She wants to
4
1
cut it into 12meter long. How many pieces of cloth will she
have?

1 3
How many 12 ‘s are there in ? (See the illustration above)
4

1 3 3 1
The figure shows that there are nine 12 ‘s in 4 or ÷ 12 = 9.
4

30
3 3 1
What is 4 x 12? Is that equal to ÷ 12? Let solve.
4

3 12 3 x 12
3 x 12 = x = = 36 = 9
4 4 1 4x1 4

3 1
We notice that dividing 4 by 12 is just like multiplying it by its reciprocal 12.

2 2
Let us divide 4 ÷ 1
6 8

26 10 Change mixed number to improper fractions


6 ÷ 8 10 8
The reciprocal of 8 is 10 .
13
26 8 4 Multiply it after you get the reciprocal of the
6 x 10 divisor.
3 5
Since it is already multiplication, you can use
13 x 4 cancellation method.
3x5
52 7
15 = 3 15

Let’s Work Together

Complete the equation then solve. Write the quotients in lowest terms.

1. 1 ÷ 5 1 1
= x =
4 4 5
2. 6 2 6 ?
÷ = x =
7 14 7 2

2 5 8
3. 5 ÷ 1 = x =
8 1 ?

4. 2 2 ? 4
8 ÷ 6 = x =
3 4 3 ?

5. ?
5 ? x
3 ÷ 23 = =
6 6 ?

31
Do on Your Own

Express each quotient in simplest form.


4 ÷ 8
1. =
7

3 2
2. ÷ =
4 3

4
3. 12 ÷ =
5

2 5
4. 6 3 ÷ =
9

5. 7 1 =
4 8 ÷ 6 4

Let’s analyze

Solve the following word problems.


1
1. How many skirts can be cut from 9 meter of cloth if each skirt needs
3
3
1 4 meters?

2. Nimfa led her classmates in making their graduation ribbons. How


3
many ribbons with length of 2 4 inches can be cut from a ribbon 1320
inches ribbon?

1
3. Carmela spent 35 minutes reading a pocketbook. If it takes her 2
3
minutes to read one page, how many pages can she read at that rate?

32
Lesson
Addition of Decimals
2.1

Name:
Level: Section: Date:
Title of the Activity:
Learning Competency: Adds decimal and mixed numbers through ten thousandths
without or with regrouping
Code: M5NS-Id-10.1

In this lesson, one will learn how to add decimals without or with
regrouping.

Objectives:

Add decimals without regrouping


Add decimals with regrouping
Solve word problems involving addition of decimals

Adding decimals is like adding whole numbers. Remember to align the


decimal points of your addends to the decimal point of your sum. When
addends have different decimal places, we annex zeros for the missing
decimal places.

Analyze this problem.


Pam has 2.4 meters of red ribbon. She bought 5.25 meters of blue
ribbon. How many meters of ribbon does she have?
The number sentence is 2.4 + 5.25 = N . Write the addends in vertical form.
Make sure that you decimal points are aligned.
Ones Decimal Point Tenths Hundredths
2 . 4 0
+ 5 . 2 5
7 . 6 5
Add from right to left.
Annex one zero to the first addend to have the same decimals place
value. Then, proceed to the addition of whole numbers.

Therefore, Pam has 7.65 meters of ribbon.

33
Let’s have another problem.

Virginia and Yolanda shared liters of paint to


make a mural for their Brigada Eskwela activity. Virginia
used 6.765 liters while Yolanda used 6.12 liters. How
many liters of paint did the two girls use?

To find how many liters, add the liters used by Virginia and Yolanda.

This will be your number sentence. 6.795 + 6.12 = N

To solve, rewrite givens in vertical form. Do not forget to align the decimal
points.
6.795
+ 6.120 Affix zero on the second addend.
12 . 9 1 5 Therefore, the two girls used 12.915 liters of paint.

Here are other examples.

Add: 4.9 + 10 + 3.12 + 0.076


4. 900 affix 2 zeros
10 . 0 0 0 affix 3 zeros
3.120 affix 1 zero
+ 0.076
18 . 0 9 6

What is the sum of 78.23 and 95.619?


78.23
+ 95.619
173.849

34
Let’s Work Together

Arrange in column and then add.

1. 0.987 + 0.0121 2. 0.23 + 0.6532


3. 1.289 + 4.786 4. 0.692 + 2.5
5. 12.67 + 87.9 6. 9.103 + 10.4
7. 8.9 + 12.76 8. 9.7 + 11.7 + 32.098
9. 0.123 + 2.12 + 89.6 10. 9.2845 + 12.767 + 267.09

Do On Your Own

Find the sum.

1. 2. 3. 4.
9.2456 89.21 8.2 9
+ 11.07 + 31.87 + 1.137 + 1.985

5. 6. 7. 8.
59.2 92.44 129.56 23.56
+ 11.7 + 22.11 + 131.97 + 21.98
22.65

35
Let’s analyze

Solve the following word problems.

1. The Boy Scouts bought the items they needed for their camping in Mt.
Makiling. They bought 58.2 kilograms of hotdog, 28.75 kilograms of
porkchop and 50 kilograms of rice. Find the total mass of the items
they bought.

2. Noel paid Php 14.75 for a notebook, Php 36.99 for a set of coloring
pencils and Php 7.65 for a ballpen. How much did he pay in all?

3. Reyna bought 0.32 kilograms of garlic, 0.75 kilograms of onion and


0.25 kilograms of pepper. How many kilograms of items did Reyna buy
in all?

36
Lesson
Subtraction of Decimals
2.2

Name:
Level: Section: Date:
Title of the Activity:
Learning Competency: Subtracts decimals and mixed decimals through ten
thousandths without or with regrouping
Code: M6NS-Id-10.2

In this lesson, one will learn how to subtract decimals without or


with regrouping.

Objectives:

Subtract decimals without regrouping


Subtract decimals with regrouping
Solve word problems involving subtraction of decimals

Like in adding decimals, we align the decimal points in subtraction of


decimals. In case the decimal places in decimal numbers are not the same,
we affix zeroes. Let us try this.

Examples:

1. Take away 64.53 from 87.56


Tens Ones Decimal point Tenths Hundredths
8 7 . 5 6
‒ 6 4 . 5 3
2 3 . 0 3
Subtract from right to left.
Align the decimal points of the given with the decimal point of the
difference

37
2. Diminish 54.82 from 95.7
Tens Ones Decimal point Tenths Hundredths
9 5 . 7 0 Affix zero
5 4 . 8 2 to the first
4 0 . 8 8 addend.

Study the problem below.

Mrs. Alcantara’s electric meter read 1349.7 on August 1 and 1480.2 on


August 30. What is the difference between these readings?

The number sentence is 1480.2 – 1439.7 = N

1480.2
‒ 1439.7
130.5 Therefore, the reading is 130.5.

At the start of a trip, Mrs. Alcantara’s car registered 2 895.7 kilometers.


At the end of the trip, it registered 3 114.8 kilometers. How far did she go?

The number sentence is 3114.8 ‒ 2895.7 = N


3114.8
‒ 2895.7
219.1 Mrs. Alcantara travelled 219.1 kilometers.

Let’s Work Together

Fill in the blank with the correct answer.


1. From the sum of 70.63 and 17.9, subtract 59.189 _________________
2. When 7.54 is taken away from a number, the answer is 4.412. Find the
number. _______________
3. Find the difference of 7.15 and 3.598. ___________________
4. Subtract the sum of 4.36 and 7.9 from 16.2 __________________
5. Subtract 46 tenths from 764 hundredths. _________________

38
Do on Your Own

Arrange in column then subtract.


1. 4.84 – 3.78 =
2. 69.74 – 53.209 =
3. 17.5 – 10.678 =
4. 234.9 – 126.2675 =
5. 10 – 2.6543 =

Solve the following problems.


1. Mother bought some meters of cloth. After she had used 6.75 meters
for pillow cases, she had 1.5 meters left. How many meters of cloth did
she have at first?

2. How much money will be left from Php 1000.00 after buying three
books that costs Php 475.50, Php305.65 and Php180.90?

Let’s analyze

Find the missing number in the equation.

1. 3.4576 2. 6.78 3.
‒ 1.8498 ‒ 4.7659 ‒ 2.3456
4.3334

4. 5. 8.9
‒ 6.9549 ‒
1.5775 4.338

39
Lesson
Multiplication of Decimals
2.3

Name:
Level: Section: Date:
Title of the Activity:
Learning Competency: Multiplies decimals and mixed decimals with factors up to 2
decimal places
Code: M6NS-Ie-13

In this lesson, one will learn how to multiply decimals.

Objectives:

Multiply decimals by whole number


Multiply decimal by another decimals
Solve word problems involving multiplication of decimals

In multiplying decimals, we multiply like whole numbers but we need to consider


the number of decimal places that we have in our factors.

Let us analyze this problem.

Ms. Lopez bakes cakes to be shared for her pupils.


She uses 0.86 kilograms of flour to bake a cake. How much
flour is needed to make 12 cakes?

We multiply 0.86 and 12 to get the number of kilograms of flour to be used.


0.86 (multiplicand) 2 decimal place
x 12 (multiplier) 0 decimal place
172
+ 86
10.32 (product) 2decimal places

Therefore, Ms. Lopez needs 10.32 kilograms of flour.

40
Another example.
A certain ribbon costs Php 8.75 per meter. How much
will 12.5 meter costs?

The number sentence is 8.75 x 12.5 =


8.75 2 decimal places
x 12.5 1 decimal places
4375
1750
875
109.375 3 decimal places

Therefore, the cost of the ribbon is Php 109.375 or Php 109.38 (when
rounded off to the nearest hundredths)

Now, you try to put the decimal point in each product correctly.
9.5 4.23 6.12 72.22 0.723
x 19 x 0.5 x 0.22 x 5.2 x 0.2
1805 2115 13464 375544 1446

What if one of our factors is a power of 10. Let study this problem.

The government uses submarine to locate objects underwater. This


submarine uses sound waves radar. Sound travels about 1.75 kilometers per
second through seawater. How far does sound travel in 10 seconds? In 100
seconds? in 1 000 seconds? in 10 000 seconds?

We multiply to find the distance travelled by the sounds.


1.75 x 10 = 17.5 1.75 x 1 000 = 1750
1.75 x 100 = 175 1.75 x 10 000= 17 500

41
What did you notice with the product? What happens to the decimal point? If
you can’t see the pattern yet, study the following.

0.54 x 10 = 5.4 1.23 x 10 = 12.3 21.0678 x 10 = 210.678

0.54 x 100 = 54. 1.23 x 100 = 123 21.0678 x 100 = 2106.78

0.54 x 1 000 = 540 1.23 x 1 000= 1 230 21.0678 x 1 000 = 21067.8

I hope you already noticed the pattern. Remember to multiply a decimal by


10, 100, 1000, etc. Move the decimal point to the right depends on the
number of zeros.

Let’s Work Together


Complete the tables. Find the product.

x 10 x 100 X 1000
0.856
4.76
0.034
95.27
123.987

x7 x 0.6 x 1.3
97
102
0.73
2.94
1.829

42
Do On Your Own
Multiply the decimals.
1. 0.009 x 8 = 6. 7 x 0.056 =
2. 7.56 x 3.6 = 7. 83.9 x 1.3 =
3. 63.9 x 2.5 = 8. 0.0065 x 7 =
4. 0.937 x 44 = 9. 20.8 x 5.54 =
5. 27.65 x 4.7 = 10. 72.65 x 6.1=

Let’s analyze

Write TRUE if the statement is correct, otherwise FALSE.

1. The product of 4 and 0.65 is less than 6.


2. The product of 3 and 2.56 is greater than 8.
3. The product of 6.753 and 6.3 has 5 decimal places.
4. The product 0.2 and 20 is 4.
5. The number of decimal points in the product depends on the
number of decimal points in the factors.

Solve the following problems.

1. Robert can run 3.75 km in an hour. How far can he run in 3 hours?

2. Antonio bought 1 dozen of pingpong balls at Php 8.75 each. How


much did he pay altogether?

43
Lesson
Division of Decimals
2.4

Name:
Level: Section: Date:
Title of the Activity:
Learning Competency: Divides whole numbers by decimals up to 2 decimal places
Code: M6NS-Ig-18.1

In this lesson, one will learn how to divide decimals.

Objectives:

Divide decimals by whole number


Divide decimal by another decimals
Solve word problems involving dividing of decimals

Division of Decimals by a Whole Number

Carlos received his monthly electric billing for the period


of 30 days. He consumed a total of 395.4 kilowatts. What is
his daily kilowatt consumption?

To get the daily kilowatt used, we divide the total kilowatts


used by the total number of days.

13.18 * Divide the decimals just like the whole numbers.


30 395.40 * Place the decimal point in the quotient directly
‒ 30 above the decimal point in the dividend.
95 * Annex one zero after 4 in the dividend and continue
‒ 90 the process of division
54
‒ 30
240 Therefore, Carlos used 13.18 kilowatts each day.
‒ 240
0

44
Here are other examples.

Divide 14.4 by 9. Divide 0.648 by 12.


1.6 0.054
9 14.4 12 0.648
‒ 9 ‒ 60
54 48
‒ 54 ‒ 48
0 0

Division of Whole Number by a Decimal

Mother bought 21 kilograms of chicken. She used


1.75 kilograms for each recipe. How many recipes did
she make out of 21 kilograms?

To solve, we to divide 21 by 1.75, to know how many recipes were made.

1 2. * In dividing decimals, the divisor must be a whole


1. ͜7͜5. 2 1 .͜ 0 ͜0 . number. If the divisor is a decimal, move the
‒ 17 5 decimal point to the right to make it a whole
number.
3 5 0 *Since you moved the decimal point in the divisor,
‒ 3 5 0 you also need to move the decimal point in your
0 dividend depending on the number of
movement in your divisor.

Here is another example.

What is 8 ÷ 0.2?
40 *Move one place to the right to make the divisor a
0.͜2 8.͜0 whole number. Annex one zero after the dividend.
‒8 * Divide like whole numbers.
0
‒0
0

45
Division of a Decimal by another Decimal

A motorcycle club rode 29.75 kilometers in 3.5 hours. How


many kilometers did they cover each hour?

To solve, we divide 29.75 by 3.5.

8.5 *Move the decimal point in the divisor and so with


3.͜5. 29.͜7.5 the dividend one place to the right.
‒ 28 0 *Put the decimal point in the quotient directly
175 above the decimal point in the dividend.
‒ 175 *Divide the numbers.
0
Therefore, they rode 8.5 kilometers each hour.

Here is another example.

Divide 146.97 ÷ 6.9

21 . 3 *Move the decimal points of the divisor and the


6.͜9. 146.͜9 .7 dividend 1 place to the right
‒138 *Put the decimal point of the quotient directly
89 above the decimal point of the dividend.
‒69 * Divide the numbers.
207
‒207
0

Let’s Work Together

Divide the input by 4 to get the output.

Input Output
36.8
46.8
131.2

46
Divide the input by 0.6 to get the output.

Input Output
5.7
19.2
30

Find the quotient.

5 3.34 1.2 6 0.3 17.4 2.5 7.85

Do on Your Own

Divide the following and compare the quotient by placing < , > or = inside the
box.

1. 30.6 ÷ 5 5 ÷ 30.6

2. 45.18 ÷ 9 4.518 ÷ 0.9

3. 8.2 ÷ 0.4 9.5 ÷ 0.5

4. 4.482 ÷ 5.4 1.15 ÷ 0.23

5. 60 ÷ 0.4 6 ÷ 0.04

47
Let’s analyze

Get the unit price of each item. Round of your answer to the nearest
hundredths if possible.

4 dozens of flowers 10 cans of corned beef


cost Php 156.00 cost Php 450.80
Price per flower is ________ Price per can of corned beef is ________

9 bars soap 4.75 kilos of pork


cost Php 294.75 cost Php 560.50

Price of one bar is _______ A kilo of pork costs __________

2.5 kilos of sugar 7 packs of coffee


cost Php 136.25 cost Php 340.55
A kilo of sugar costs ________ A pack of coffee is ________

48
Lesson
Concept of Ratio
3.1

Name:
Level: Section: Date:
Title of the Activity:
Learning Competency: Expresses one value as a fraction of another given their ratio
and vice versa
Code: M6NS-IIa 26

In this lesson, one will learn about the ratio.

Objectives:

Define and illustrate the meaning of ratio using concrete or pictorial


models
Express one value as a fraction of another given ratio and vice versa
Find how many times one value is as larger as another given their ratio
and vice versa

For every 2 pieces of toasted bread Zeny


bought, she paid the cashier ₱ 7.00. How much she
paid for 10 pieces of toasted bread?

Zeny used a ratio to compare the pieces of


toasted bread she bought with the amount she paid
in pesos. A ratio is a comparison of two quantities.
The ratio of toasted bread to pesos is 2 is to 7. You
can write a ratio in several ways.
2
2 is to 7 2:7 7
(word form) (colon form) (fraction form)

All these ratios are read as “two is to seven.” Remember that order is
important.
The ratio of pesos to toasted bread is 7 is to 2.
7
7 is to 2 7:2 2

49
From the picture above we also see that for every 4pcs of toasted bread,
Laurenz paid ₱14.00; for 6 toasted bread ₱21.00; for 8 toasted bread ₱28.00;
and for 10 toasted bread ₱35.00. The picture suggested equivalent ratio for 2
is to 7:

2 4 6 8 10
= 14 , 21 , 28 , 35
7

You can find equivalent ratios by thinking equivalent fractions. Can you
give other ratios that are equivalent to the ratios above?

Ratio must be expressed in simplest form.

In a class of 54 pupils, 24 are girls. What is the ratio of boys to girls?

30
The ratio of boys to girls is 30 is to 24. It may also be written as 30:24 or .
24
30 6𝑥5 5
Since the ratio is not yet in lowest term, we can express it as: 24 = 6 𝑥 4 = 4

What is the ratio of square to circle? ______ : ______


What is the ratio of circle to triangle? ______ : ______
What is the ratio of triangle to square? ______ : ______

Here is another example.

What is the ratio of vowels to consonants in the word “MATHEMATICS”?

The vowels are A, E, I, therefore there are 3 vowels. The consonants


are M, T, H, C, S, therefore there are 5 consonants. Since there are 3 vowels
and 5 consonants in the word “MATHEMATICS”, the ratio is 3 is to 5 or 3:5 or
3
.
5

50
To compare two measurements, make sure that the numbers are of the
same units. For example:

Compare the mass of a 0.1 kilogram of book with the mass of a 15-gram
pen. What is the ratio of the mass of the book to the mass of the pen?

𝑚𝑎𝑠𝑠 𝑜𝑓 𝑏𝑜𝑜𝑘 0.1 𝑘𝑔 100 𝑔


= = = 100:15 = 20:3
𝑚𝑎𝑠𝑠 𝑜𝑓 𝑝𝑒𝑛 15 𝑔 15 𝑔

Let’s Work Together

Write the ratio in colon form then in fraction form.

Ratio Colon Form Fraction Form


1. 3 plates to 9 spoons
2. 4 carrots to 10 cabbage
3. 12 boys to 15 girls
4. 8 pens to 16 papers
5. 5 Math books to 7 Science
books

Express the ratio in lowest term.

1. 20 : 25 6. 9 trays to 45 glasses
2. 18 : 24 7. 12 books to 15 pupils
3. 30 : 100 8. 3 minutes to 100 seconds
4. 12 is to 36 9. 2 kilograms to 200 grams
5. 18 is to 12 10. 6 months to 1 year

51
Do on Your Own

Use the figures to answer the following questions. Simplify your answer if
possible.

1. What is the ratio of thunder bolts to crescent moons?


2. What is the ratio of suns to crescent moons?
3. What is the ratio of suns to thunder bolts?
4. What is the ratio of crescent moons to the total figures?
5. What is the ratio of thunder bolts to crescent moons to suns?

Reduce the ratio to its simplest form.

1. 4 : 10 6. 16 slippers to 8 children
2. 9 : 27 7. ₱50 to 10 guavas
3. 28 : 21 8. 8 kilograms to 1200 grams
4. 9 is to 24 9. 5 minutes to 200 seconds
5. 32 is to 14 10. 2 years to 10 months

52
Let’s analyze

Study the table then answer the following questions. Simplify your answers
whenever possible.
Favorite Sports of Pupils

20

15

10

0
basketball volleyball badminton tennis

1. How many pupils answered the survey?


2. What is the ratio of basketball to volleyball?
3. What is the ratio of tennis to badminton?
4. What is the ratio of basketball to total pupils?
5. What is the ratio of volleyball and tennis to total pupils?

Solve these word problems.


1. There are 36 dove birds and 30 maya birds in an aviary. State the ratio
of maya birds to dove birds in simplest form.

2. In an orchard, there are 32 banana trees, 30 guava trees, 18 avocado


trees and 15 papaya trees. State the relation of each tree in simplest
form.
a. Banana trees to papaya trees
b. Guava trees to avocado trees
c. Guava trees to all trees in orchard
d. Banana trees to guava trees to avocado trees to papaya trees

53
Lesson Concept of Proportion
3.2

Name:
Level: Section: Date:
Title of the Activity:
Learning Competency: Set up proportions for groups of objects or numbers
Code: M6NS-IIa 29.1

In this lesson, one will learn about proportion.

Objectives:

Define and illustrate proportion using concrete or pictorial models.


Set up proportions for groups of objects or numbers and for given
situations.
Find a missing term in a proportion.

Cora’s height is 5 ft and her shadow is 4 ft, while the lamppost height is 15
ft and the shadow of the lamppost is still unknown. In order to measure the
shadow of lamppost, we have to apply proportion.

Solution 1: Fraction Form:

𝐶𝑜𝑟𝑎′ 𝑠 ℎ𝑒𝑖𝑔ℎ𝑡 𝑙𝑎𝑚𝑝𝑝𝑜𝑠𝑡′𝑠 ℎ𝑒𝑖𝑔ℎ𝑡


= 𝑙𝑎𝑚𝑝𝑝𝑜𝑠𝑡′𝑠
𝐶𝑜𝑟𝑎′ 𝑠 𝑠ℎ𝑎𝑑𝑜𝑤 𝑠ℎ𝑎𝑑𝑜𝑤
5 15 Using the fraction form, we need
=
4 𝑥 to do cross multiplication. Divide
5x = (4)(15) both side by 5.
5𝑥 60
=
5 5
x = 12 ft

54
Solution 2: Colon Form:
Cora Lamppost Using the colon form, we
height : shadow = height : shadow need to get the product of
5 : 4 = 15 : x means and divide it to the
means given extreme.
extremes (4)(5) = 60
Therefore, the shadow of lamppost is 12 ft. 60 ÷ 5 = 12 ft
Now you try to solve for the missing term.

3 x 𝑥 4
= 24 =9 5 : x = 25 : 30 2 : 7 = 18 : x
8 36

When two ratios are equal, they are proportion. A proportion is an


equation that states the equality of two ratios. Each part of a proportion is a
term. The two end numbers (first and last terms) are called extremes and the
two middle numbers (second and third terms) are called means. If the products
of means and extremes are equal, then it is proportion.

Is the ratio 3:4 equal to, greater than or less than 9:12?

Solution 1: Using fraction form

3 9
= Write as fractions
4 12
(3)(12) = (4)(9) Cross multiply
36 = 36 Since the cross product is equal, the ratios are
equal, then it is proportion.

Solution 2: Using colon form

3 : 4 = 9 : 12 Get the product of means.


(4)(9)=36 Get the product of extremes.
(3)(12)=36 The ratios are equal, then, they are
proportion.

Which of the following is a proportion?

3 6 2 5
1. = 2. = 3. 4:3 = 20:15 4. 10:11 = 5:6
4 9 8 20

55
Let’s Work Together
Use the following data.

4 yellow bells, 5 sampaguitas, 6 sunflowers, 8 roses, 3 orchids

Are the following equations proportion? Encircle your answer.


1. orchids to sunflowers = yellow bells to roses YES NO
2. sampaguitas to yellow bells = orchids to sunflowers YES NO
3. roses to yellow bells = sunflowers to orchids YES NO
4. yellow bells to sunflowers = orchids to roses YES NO
5. orchids to sampaguitas = orchids to yellow bells YES NO

Solve for x to get proportion.


1. 2:3 = x:9
2. x : 4 = 12 : 16
3. 5 : x = 20 : 30
𝑥 18
4. =
30 20
21 14
5. =
𝑥 16

Do On Your Own

Determine whether the two ratios are proportion or not. Write (=) or (≠) in the
circle.
1. 4:7 12 : 21
2. 1:2 6 : 12
3. 6:8 7 : 9
3 9
4. 15 45
9 3
5. 16 8

56
Solve the following

1. 5 : n = 30 : 48
2. 16 : 18 = n : 9
5 𝑛
3. = 56
8
𝑛 2
4. =3
21

5. Is the ratio 5 is to the same as 2.5 is to 1?

Let’s analyze

Create a proportion from each set of numbers. Use the number once.

Example: 1. 2.
4, 2, 10, 12, 6 2, 12, 5, 16, 30 10, 40, 20, 8, 4

4 : 2 = 12 : 6 ------------------- --------------------

3. 4. 5.
5, 20, 22, 11, 44 3, 5, 7, 21, 35 28, 18, 24, 7, 6

------------------- --------------------- ---------------------

Solve the problems.

1. Fifteen test tubes to three test tube racks. How many test tubes for 4
racks?

2. Two Math books for 3 children. How many books for 6 children?

3. Onions are sold for ₱5.00 for 2 pieces. How much you will pay for a
dozen?

57
Lesson
Types of Proportion
3.3

Name:
Level: Section: Date:
Title of the Activity:
Learning Competency: Sets up proportion for a given situation or problems.
Code: M6NS-IIa 29.2

In this lesson, one will learn about the different kinds of proportion.

Objectives:

Solve problems involving direct proportion, inverse proportion, and


partitive proportion.
Create problems involving ratio and proportion with reasonable
answers.

1. Direct Proportion ( ↓↓ , ↑↑ ) shows that an increase in one quantity causes


a corresponding increase in the other quantity or a decrease in one
quantity causes a corresponding decrease in the other quantity.

These are some of the situations showing direct proportion.

a. The faster the speed, the longer distance covered.


b. The bigger the principal, the bigger the interest.
c. The taller the object, the longer the shadow.
d. The greater the width and length, the greater the area.

58
Example:
A car travels a distance of 90 kilometers in 2 hours. How many
kilometers will it cover for 6 hours?

Solution:
To solve, we need to make a proportion. The 2 quantities being
compared are distance (kilometers) and time (hours).
kilometers : hours = kilometers : hours
90 : 2 = x : 6
2x = (90)(6)
2𝑥 540
=
2 2
x = 270 kilometers
Therefore, the car travelled 270 kilometers for 6 hours.

2. Inverse Proportion/Indirect Proportion (↓↑ , ↑↓) shows that an increase in


one quantity causes a corresponding decrease in the other quantity or a
decrease in one quantity causes a corresponding increase in the other
quantity.
These are some of the situations showing inverse/indirect proportion.
a. The more contestants in a contest, the lesser chance to win.
b. The more men working, the shorter time to finish the work.

Example:
If 8 men can consumed a pack of food for 6 days, how long would the
food last for 12 men?

Solution:
The problem is an inverse proportion because the food will last
shorter for more men. Let the number of days be x.
First quantity Second quantity
We have 6 days for 8 men,
x days for 12 men
Let us make a proportion.
6 12
=
𝑥 8
12x = (6)(8)
12𝑥 48
=
12 12
x = 4 days
Therefore, the pack of food can last for 4 days if there were 12 men.

59
3. Partitive Proportion is used to divide a whole into parts proportional to the
given ratio.

Example:
Thirty – six liters of oil are transferred into 3 oil canisters in the ratio
1:2:3. How much oil is in each canister?

Solution:
The ratio 1:2:3 means 1 + 2 + 3 = 6 parts. 36 liters will be divided into
6 parts.

36 ÷ 6 = 6 liters per part

Canister 1 has 1 part, therefore 1 x 6 = 6 liters.


Canister 2 has 2 parts, therefore 2 x 6 = 12 liters.
Canister 3 has 3 parts, therefore 3 x 6 = 18 liters.

Therefore, canister 1 has 6 liters in it, canister 2 has 12 liters in it and


canister 3 has 18 liters in it.

Let’s Work Together

Identify if the problem involves direct, inverse or partitive proportion. Then solve
the problems.
1. Three bananas cost ₱12.00. What is the cost of 1 banana?
2. Five pupils take 3 minutes to arrange the books in the shelf.
How long will it take 2 pupils to do it?
3. John Erick’s allowance can last in 12 days if he spends
₱20.00 a day. How long will it last if he spends ₱30.00 a
day?
4. There are 36 pupils in a class. The ratio of boys to girls is
5:4. How many boys are there?
5. Four books cost ₱1 300.00. What is the cost of 20 books?

60
Do on Your Own
Solve the following word problems.

1. If 13 kilograms of chicken cost ₱1560, calculate:


a. How much is the cost of 18 kg of chicken?
b. How many kilogram of chicken can be bought for ₱180?

2. Two numbers are in the ratio of 3:7. The sum of the two numbers is 340.
a. What is the bigger number?
b. What is the smaller number?

3. If 4 men can weed the garden in 42 hours. How long will 6 men do the
job?

4. At the rate of 3 items for ₱45.00, how many items can you buy if you
have ₱150.00?

5. The ratio of the angles of a triangle is 2:4:6. What is the measure of each
angle?

Let’s analyze

Create a problem using the following information.

1. Direct proportion; ratio is 3:4:9: n

2. Inverse proportion; 8 men for 9 days, 12 men for n days

3. Partitive proportion; ratio is 2:3:4, total amount ₱450.00

61
Lesson
Understanding Percent
4.1

Name:
Level: Section: Date:
Title of the Activity:
Learning Competency: Solves routine problems involving finding the percentage,
rate and base using appropriate strategies and tools
Code: M6NS-IIc 34.1

In this lesson, one will understand percent. Finding and solving the
percentage, base or rate in a given situation.

Objectives:

Define and illustrate the meaning of percent using concrete or pictorial


models.
Find the percentage, rate, or percent in a given problem.

 is a number or ratio expressed as a fraction of 100


 means parts per hundred
Percent  comes from the Latin per centum, which means per
hundred
 use the symbol %
 can be express a fraction or decimal

62
Illustrating the meaning of percent by giving some examples.

Example 1:

Out of hundred squares, there are 20 squares with a heart inside.


20 % means the ratio of 20 to 100.

Example 2:
Write the ratio 61 to 100 in percent form.
The ratio 61 to 100 is equivalent to 61 per 100 or 61%.

Example 3:
43 items correct out of 100.
43
= 43%
100

Changing Percent to Decimal


Examples: Write each percent in decimal form.
1 1
a. 19% b. 7.5% c. 46 % d. %
3 4

Solution:

*If the decimal point is not shown , it is always understood to be at the right of
the whole number. Move two places to the left, add zeros as place holders if
necessary then remove the percent symbol.

a. 19% = 0.19
b. 7.5 % = 0.075
1
c. 46 3 % = 46. 3333…. % = 46.33% = 0.4633
1
d. % = 0.25 % = 0.0025
4

63
Changing Decimal to Percent

Examples: Write each decimal in percent.

a. 0.7 b. 0.85 c. 31.5 d. 2.016

Solution:

*To change decimal to a percent, move the decimal point two places to the
right, and add the percent symbol.

a. 0.7 = 70%

b. 0.85 = 85%

c. 31.5 = 3150%

d. 2.016 = 201.6%

Changing Percent to Fraction

Examples: Write each percent in fraction form.

a. 15% b. 50% c. 61%

Solution:

*To change percent to fraction, drop the % symbol and make 100 as the
denominator. Always express the fraction in simplest form.

15
a. 15% = 100

50 1
b. 50% = 100 = 2

61
c. 61% = 100

64
Changing Fraction to Percent
Examples: Write each fraction in percent form.
1 3 5
a. b. c.
4 8 6

Solution:

*To change fraction to percent, multiply the given fraction by 100%.

1 1
a. = x 100% = 25%
4 4

3 3 1
b. = x 100% = 37 2 % or 37.5%
8 8

5 5 1
c. = x 100% = 83 % or 83.33%
6 6 3

Let’s Work Together

A. Write each ratio in percent form.

1. 85 voters out of 100


2. 25 items correct out of 50
3. 12 students out of 48
4. 44 parents out of 100
5. 15 books out of 45

B. Express each in percent form.

1. 0.23
48
2. 100
3. 17.6
4
4. 5
5. 0.875

65
Do on Your Own
Find the missing form.

Percent Decimal Fraction


1
1. __________ __________ 2

2. __________ 0.75 __________


3. 65% __________ __________
4. 120% __________ __________
5. __________ 0.7 __________
1
6. __________ __________ 122
3
7. _________ __________ 20

8. 23.5% __________ __________


9. __________ 3.2 ___________
1
10. __________ __________ 12

Let’s Analyze

Solve each word problem.

1. A school guard has been doing rounds in the school 70% of the time he
is on duty. What fraction of his time does the school guard spend in
making rounds?

2. One-third of 45 students go to school by school bus, 20 ride by their own


car, and the rest are taking public vehicle. What percent of the class
takes the public vehicle?

66
Lesson
4.2 Percentage, Rate and Base

Name:
Level: Section: Date:
Title of the Activity:
Learning Competency: Solves routine problems involving finding the percentage,
rate and base using appropriate strategies and tools
Code: M6NS-IIc 34.1

In the previous lesson, we learned to illustrate percent, how to


change percent to decimal, percent to fraction and vice versa. This
time, one will find the percentage, rate and base using
appropriate examples.

Objective:

Solve routine and non-routine problems involving finding the


percentage, rate and base using appropriate strategies and tools.

Let’s try the example

75% of 120 participants were students. How many of the participants are
students?

Form an equation 75% of 120 is what number?


75% x 120 = n
0.75 x 120 = n Express the percent as a decimal
90 = n

Therefore 90 out of 120 participants are students.

Using the derived formula:


P= Rx B
P – Percentage
R - Rate
B - Base

- Rate

67
Finding the Rate

Let’s try the example

Nica’s monthly allowance is ₱4000. If she spends


₱2500 for the food and transportation. What percent
of her earnings is spend for the food and
transportation?

Form an equation n x 4000 = 2500


2500 5
n = 4000 = 8 divide or simplify
n = 0.625 x 100%
n = 62.5%

Using the derived formula: 𝑷


R= X 100%
𝑩
P – Percentage
R - Rate
B - Base
Finding the Base
- Rate
Let’s try the example

The Schools Division of Parañaque City conducted a seminar for teachers.


There were 250 teachers who attended the seminars. If these represent 40%
of the total participants, how many participants are there in all?

Form an equation 250 is 40% of what number?


250 = 40% of n
250 = 0.4 x n
250
n =
0.4
n = 625 participants

Using the derived formula:

𝑷
B= 𝑹
P – Percentage
R - Rate
B - Base

- Rate

68
Let’s Work Together
Use the equation P = B x R to answer each question.

1. What is 9% of 360? 6. 25 is what percent of 50?


2. What is 12% of 150? 7. 12% of what number is 36?
3. What is 36% of 50 8. 64 is 8% of what number?
4. What percent of 50 is 20? 9. 30% of what number is 24?
5. 15 is what percent of 75? 10. What number is 50% of 80?

Do on Your Own

State an equation involving a variable that expresses the condition of the


question.

1.40% of what number is 90? 6. 1.2 is what percent of 20?

2.40% of 25 is what number? 7. 125% of what number is 85?


1
3.What percent of 75 is 36? 8. 52 2 % of 300 is what number?

4.What number is 15% of 64? 9. 35 is out of 120 is what percent?

5.45 is 30% of what number? 10. What percent of 80 is 100?

Let’s Analyze

Solve each word problems.


1. Cesar answered 50 items in a Math quiz. If 38 of these are correct, how
many percent did he got a correctly?
2. Elsa earns ₱40,000 a month. She saves 35% of it. How much does
she save in a month?
3. Jeremy’s record shows that he was absent for 2 weeks out of the total
number of school days. If these represents 5% of the total number of
school days, what is the total number of school days in one school year?

69
Lesson Solving Word Problems Involving
4.3 Percent

Name:
Level: Section: Date:
Title of the Activity:
Learning Competency: Solving non routine problems involving finding the
percentage, rate, and base using appropriate strategies and tools
Code: M6NS-IIc 34.2

In the previous lesson, one learned to find percentage, rate and


base. This time, we will solve problems involving percent.

Objective:

Solve percent problems such as percent increase/percent decrease,


(discount, original price, rate of discount, sale price, marked up price)
commission.
Create problems involving percent with reasonable answers.

Why do we need to solve problems involving percent? The problems can be


use in real life situations such as in computing discounts, taxes, and
commissions.

Formula:

𝐴𝑚𝑜𝑢𝑛𝑡 𝑜𝑓 𝑖𝑛𝑐𝑟𝑒𝑎𝑠𝑒/𝑑𝑒𝑐𝑟𝑒𝑎𝑠𝑒
Percent of increase/decrease = x 100
𝑂𝑟𝑖𝑔𝑖𝑛𝑎𝑙 𝑃𝑟𝑖𝑐𝑒

70
Example 1:

A bag that costs ₱600 last month is now at ₱900.


What is the percent of increase?

Solution:
900−600
Percent of increase = x 100
600

300
= 600 x 100

= 0.5 x 100

= 50%

Example 2:

A blouse is on sale from the original cost of ₱1500 to ₱975. What is the percent
of decrease?

Solution:
1500−975
Percent of decrease = x 100
1500

525
= 1500 x 100

= 0.35 x 100

= 35%

Definition

Discount – refers to the reduction in prices.


Marked Price / Original Price – is the regular price or list price.
Sale price or net price - the price of an article after the discount has been
deducted.
Discount Rate – when a discount is stated as a percent number.

71
Example 3:

The marked price of a book is ₱120. How much is the selling price if 20%
discount is given to those who buy more than 10 copies?

Solution:

₱120 marked price, 20% rate of discount Selling price = 120 – 24


Discount = 20% of 120 = ₱96
= 0.2 x 120
= ₱24

Example 4:

What rate of discount is given if a blouse that regularly sales for ₱100 is sold
for ₱60?

Solution: ₱100 is the marked price, ₱60 is the sale price


100 – 60 = ₱40 is the discount
40
Rate of discount = 100 x 100% = 40%

FORMULA:

D = MP X r% where:
SP = MP – D D – discount
r % = D ÷ MP MP – Marked price
𝑺𝑷
MP = SP – Selling price
𝟏𝟎𝟎% − 𝒓%
r% - Rate of discount

DEFINITION

A sales tax is a tax that is added to the price of goods and services. The total
price is equal to sales tax plus the original price.

Example 5:

Neneng purchased a pen for ₱35. She needs to pay a sales tax of 10%. Find
the amount of tax and the total price.
72
The formula in finding the total price:

Purchase Price + Amount of Tax = Total Price

sale tax or amount of tax = 10% of ₱35

= ₱3.50

Therefore, the total price is (₱35 + ₱3.50) = ₱38.50

DEFINITION

Commission is a percentage of the total sales earned or received for a


business transaction.
To compute for commission we use the formula:
Commission = Rate of commission x total sales
𝑐𝑜𝑚𝑚𝑖𝑠𝑠𝑖𝑜𝑛
Rate of commission = 𝑡𝑜𝑡𝑎𝑙 𝑠𝑎𝑙𝑒𝑠 x 100
𝑐𝑜𝑚𝑚𝑖𝑠𝑠𝑖𝑜𝑛
Total sales = 𝑟𝑎𝑡𝑒 𝑜𝑓 𝑐𝑜𝑚𝑚𝑖𝑠𝑠𝑖𝑜𝑛
Total income = Total Sale – Commission

Example 6:

Sonia received a 20% commission after selling a memorial plan for ₱60000.
What was her commission?

Solution:

To find Sonia’s commission, simply multiply the selling price by the rate of
commission.Therefore, Sonia’s commisision is

20% of ₱60000 = 0.2 x 60000 = ₱12000

73
Let’s Work Together

A. Complete the table.


Original New Increase or Amount of Percent of
Amount Amount Decrease Increase or Increase or
Decrease Decrease
1. ₱40 ₱55
2. ₱130 ₱80
3. ₱75 ₱125
4. ₱100 ₱150
5. ₱250 ₱200

B. Find the sale price or total price after applying the given condition.
1. Polo Shirt: ₱ 780
Discount: 10%
2. Cellphone: ₱15000
Sales Tax: 5%
3. Notebook: ₱12.50
Sales Tax: 15%
4. Electric Fan: ₱975
Discount: 25%
5. Wrist watch: ₱7500
Sales Tax: 20%

Do on Your Own

A. Complete the table.

Marked Price Discount Rate Discount Sale Price


1. 10% 50
2. P650 P325
3. 20% P450
4. P 99.99 P2.50
5. P240 P1760

74
B. Solve each of the following under the following condition.

1. 6% commission in ₱12,000 total sales


2. 10% commission in ₱350 total income
3. ₱500 commission in ₱20,000 income
4. ₱400 commission in ₱30,000 total sales
5. ₱1,500 commission at 25% rate

Let’s Analyze
Solve each of the following problems.

1. A shirt marked ₱225 is sold for ₱150. Find the percent of discount.

2. Find the discount on a pair of shoes with a sale price of ₱499.95 at a


discount rate of 30%?

3. An agent will received 20% commission for every sold items. If an


agent sold a total sales of ₱15,500 worth of goods, how much would
an agent get?

4. The list price of a cellphone is ₱8,700. It is offered on sale at 12%


discount. How much is the discount? the sale price?

5. Cedric receives a 15% commission on magazine subscriptions. After


one week her sales totaled ₱ 8,600. What is his commission for the
week?

75
Lesson The Set of Integers
5.1 and the Number Line

Name:
Level: Section: Date:
Title of the Activity:
Learning Competency: Represents integers in the number line
Code: M6NS-IIg-44

In this lesson, one will learn the relation of integers in a real-life


situations and represent them through a number line.

Objectives:

Identify real-life situations that make use of integers.


Describe the set of integers.
Compare integers with whole numbers, fractions and decimals.
Represent integers on the number line.
Compare and arranges integers.

Integers is a solution in a problem that cannot be solved by a whole number


as 5 – 8. This problem needs a number to represent answer less than zero.

Then, what is an Integer?

An Integer is a set of numbers which consists of positive numbers, zero and


negative numbers. It is the union of the set of whole numbers and negative
numbers.

I = { …, -3, -2, -1, 0, 1, 2, 3, …}

Integers can be applied in real life situations through the concept of


oppositeness. Opposites can be represented to integers.

A deposit of ₱5000 in a bank means +5000, which is the opposite of making


a withdrawal of ₱5000, means -5000.

76
Other examples are:

1. An increase of 15 kg means +15 while a decrease of 15 kg means -15.


2. 12 meters going up means +12 while going down 12 meters is -12.
3. A rise of 70C in temperature is +7 and a drop of 70C means -7.

Integers can also be illustrated through a number line. In a number line,


zero (0) is the reference point. It is neither positive nor negative. Numbers
to the right of zero are positive numbers while to the left of zero are the
negative numbers.

Positive numbers or counting numbers are greater than zero and negative
numbers are less than zero.

The number line can be used to determine the value of the integer. Any
number to the right of a given number is greater than that number and any
number to the left of a given number is less than to that number.

Example 1:

Which is greater -2 or 3 ?

Solution:

By looking at the number line, 3 has a greater value.


Since 3 is on the right of -2, then 3 > -2 or -2 is on the left of 3 means -2 < 3.

77
Example 2:
Compare -4 and -1
Since -4 is on the left of -1, then -4 < -1 or -1 > -4.
Example 3:
Arrange the integers -5, -3, 4, 0, and 1 in increasing order.
Solution:
Plot the integers on the number line and write as they appear from left to right
as: -5, -3, 0, 1, 4

Let’s Work Together

A. Represent the following situation in integer.


1. Walking 10 blocks south
2. An increase of ₱20 in a daily allowance
3. 550 meters below sea level
4. A deposit of ₱1500
5. A gain of 18 kg
B. Give the opposite of the following.
1. Decrease
2. Left
3. West
4. Upward
5. profit
C. Plot the following set of integers on the number line.
1. { 2, 6, 0, 10}
2. {-4, -2, 0, 5, 7}
3. {integers between -3 and 3}
4. {integers from -5 to 2}
5. {positive even numbers greater than -2 but less than 10}

78
Do on Your Own

A. Write the opposite of the following integers.


1. 15
2. -6
3. -23
4. 11
5. -48

B. Complete the table.

Situation Representation Opposite of Representation


of the situation the situation of the opposite
a gain of 6 kg +6 a loss of 6 kg -6
1. 10 meters to the right
2. a deposit of ₱4500
3. a loss of ₱50
4. an increase of 3 kg
5. 15 ft. below sea level

C. Write the integers that is represented by each point.

F A G H B I E C J D

1. A 6. F
2. B 7. G
3. C 8. H
4. D 9. I
5. E 10. J

79
Let’s Analyze

A. Choose the greater number in each pair of integers.

1. -6 ; -12
2. 0 ; 10
3. 23 ; 45
4. -9 ; -1
5. -16 ; 16

B. Compare the integers. Write < or > in the blank.

1. -24 _____ -5
2. -45 _____ 0
3. 8 _____ 2
4. -1 _____ -9
5. -24 _____ 24

C. Arrange the following integers in increasing order.

1. 0 , -5, -8, 4, 5
2. -4, 6, -6, -1, 10
3. -3, -1, 7, 6, -8
4. 28, -28, -6, -3, 4
5. -3, -8, -10, -9, -1

80
Lesson
Addition and Subtraction of Integers
5.2

Name:
Level: Section: Date:
Title of the Activity:
Learning Competency: Performs addition and subtraction of integers
Code: M6NS-IIh 47.1, M6NS-IIh 47.2

In the previous lesson, we learned to describe integers and


represents them in a real life situation. This time, one will find
the sum and difference of two or more integers using different
methods.

Objectives:

Describe and interpret addition and subtraction of integers using


materials such as algebra tiles, counters, chips and cards.
Perform addition and subtraction of integers.
Solve routine and non-routine problems involving addition and
subtraction of integers using appropriate strategies and tools.

There are different ways to add and subtract integers like using materials
such as algebra tiles, counters, chips and cards. We can also use a number
line.
Simply remember the rules in addition and subtraction of integers.

Addition of Integers

Rules in Adding Integers


1. To add integers with the same signs, add their absolute value and copy the
common sign.
Examples:
a. 3 + 5 = │3│ + │5│
= 3+5
=8
b. (-4) + (-6) = - │-4│ + │-6│
=-(4+6)
= - 10

81
2. To add integers with different signs, subtract their absolute values and
copy the sign of the integer with the greater absolute value.

Examples:

a. (-12) + 8 = │-12│ + │8│


= 12 – 8
=-4
b. (-9) + 15 = │-9│ + │15│
= 15 – 9
=6

Another way of finding the sum of integers is using the algebra tiles.

+ -

A blue tile represents the positive numbers while the red tile represents
the negative numbers.

Remember, if we add 1 blue tile + to 1 red tile - , it is always


equal to zero it is called zero pair.

In symbols,
1 + (-1) = 0 + -
(-1) + 1 = 0
- +
Examples:

a. 2 + 3 = 5 + + + + + +

b.(-5) + ( -1) = -6 - - - - - + -

c.(-2) + 4 = 2 - - + + + + +

d.2 + (-3) = -1 + + + - - -

82
This time, let us use a number line in adding integers.

Examples:
a. What is 1 + 3?

Through the number line, we start from 1 unit and move 3 units to the right
since we are adding a positive number. Therefore, 1 + 3 = 4.

b. Add -4 and -5.

From -4 , we move 5 units to the left since we are adding a negative


numbers. So, (-4 ) + (-5) = -9

Subtraction of Integers
Since we know that Subtraction is the opposite of Addition then we can
state the rule in subtraction as adding the opposite of the subtrahend to the
minuend. In symbols, a - b = a + (-b) , where a represents the minuend and b
represents the subtrahend.

Examples:
1. 9–2 = 9 + (-2)
=7
2. 8 – (-4) = 8 + 4
= 12
3. (-10) – (-3) = (-10) + 3
= -7
4. (-12) – (-15) = (-12) + 15
=3

Just like in addition, we can also use an algebra tiles and number line in
subtracting integers.

Examples:
1. 7 – 4 = 3 + + + + + + +2.
Simply take away 4 blue tiles from 7 blue tiles then the remaining 3 blue
tiles is the result.

83
Another solution is through the number line,

Since -4 is the opposite of 4 which is the subtrahend, from 7 units we will


move 4 units to the left. So, 7 – 4 = 3

2. (-5) – (-2) = -3
- - - - -

From 5 red tiles, remove 2 red tiles. The remaining 3 red tiles is the result
of subtracting -2 from -5.

Same thing in the number line, from -5 move 2 units to the right since -2 is
the subtrahend.

3. 3 – (-4) = 7 + + + + + + +

- - - -
On this example, we add 4 zero pairs in order to remove 4 negative tiles.
The result is 7.

In the number line, from 3 units we move 4 units to the left since -4 is
the subtrahend.
4. 1 – 3 = -2 + + +

- -

84
Since a bigger number, 3 is subtracted from a smaller number 1, we will
add 2 zero pairs to take away 3 blue tiles. The result is -2 represented by 2 red
tiles.

In this example, from 1 unit we move 3 units to the left since the subtrahend
is 3 and its opposite is -3.

Let’s Work Together

A. Give the sum of the following using the algebra tiles.


1. (-1) + (-5) 6. (-1) + 3
2. 9 + (-12) 7. 10 + 5
3. 15 + (-17) 8. (-9) + (-2)
4. (-6) + 8 9. (-8) + 4
5. (-2) + (-9) 10. 4 + (-3)

B. Find the difference of the following using the number line.


1. (-15) – 6 6. (-13) – (-1)
2. 2 – (-9) 7. 16 – 5
3. (-8) – (-10) 8. (-14) – (-5)
4. 12 – 8 9. 25 – 16
5. (-14) – 7 10. (-16) –

Do on Your Own
Perform the indicated operation.
1. -45 + 63 11. 0 – 56
2. (-95) + (-2) 12. 48 – 65
3. (-60) + (-56) 13. (-15) - 22
4. 55 + 42 14. (-35) – 10
5. (-78) + 78 15. (-35) – 10
6. 105 + 201 16. (-125) – (-3)
7. (-18) + (-12) 17. (-63) – (-60)
8. 36 + (-28) 18. 35 - (-35)
9. 42 + 16 19. (-21) - (-9)
10. 29 + (-89) 20. 27 - (-12)
85
A. Simplify

1. (-2) + 5 - (-9) 6. 10 - (-4) + (-1)


2. (-15) + 15 + 14 + (-14) 7. (-24) + 6 - 1
3. 40 + (-10) + (-2) + 7 8. (-9) + (-2) - (-5)
4. 5 + 2 - 16 9. 22 + 16 + 14 + 5
5. 9 - (-6) + (-2) 10. (-15) - 2 + 6

Let’s Analyze

Use integers to solve each problem.

1. Sophia borrowed ₱55 on Monday and returned ₱25 pesos on Friday.


How much did Sophia still owe?

2. What is the result of borrowing ₱1500 then paying ₱800 ?

3. An ice dealer sold a total of 105 sacks of ice in April, 95 sacks in May ,
and 70 sacks in June. How many were sold in three consecutive
months?

4. The enrolment of a certain school in Parañaque at the beginning of the


school year was 12 436. After two months, there were 62 who dropped
out but 38 transferred in. During the semestral break 12 more students
were admitted and 26 more students dropped. What is the enrolment at
the beginning of the second semester?

5. The price of a kilogram of rice increased by ₱5 last year and decreased


by ₱3.75 this year. Find the difference between the increase and the
decrease in price?

6. The temperature at 8:00 a.m. was 230C. The temperature dropped by


30C at 4:00 p.m. What was the temperature at 4:00 p.m.?

86
Lesson
Multiplication and Division of Integers
5.3
Name:
Level: Section: Date:
Title of the Activity:
Learning Competency: Performs multiplication and division of integers
Code: M6NS-IIh 47.3, M6NS-IIh 47.4

In previous lesson, one learned how to add and subtract integers


using the algebra tiles and number line. Now, we will discuss
multiplication and division of integers.

Objectives:
Lesson
Describe and interpret multiplication and division of integers using
5.3
materials such as algebra tiles, counters, chips and cards.
Perform multiplication and division of integers.
Solve routine and non-routine problems involving multiplication and
division of integers using appropriate strategies and tools.

Multiplication of Integers

We know that multiplication is the short cut of repeated addition. Then,


2 x 4 means 2 + 2 + 2 + 2 or 4 x 2 in addition is 4 + 4 .

Can we use a number line or algebra tiles in this multiplication problem


2 x 4 or 4 x 2?

Let’s try.
+ + + + + + + +
2 x 4 = 2+ 2 + 2 + 2

87
Based on the illustration, 2 x 4 = 8. In multiplying a positive number by
a positive number it gives a positive product.

What if we have, 3 x (-3)? In addition, 3 x (-3) means (-3) + (-3) + (-3).

Through an algebra tiles,

3 x (-3) = -9 - - - - - - - - -

Simply count 3 groups of -3 which gives -9 result. So, if we multiply a


positive number by a negative number its product is negative.

What is the result of multiplying a negative number by a negative


number?

We can show this using a number pattern.

From the example, (-3) x 3 = -9

(-3) x 2 = -6

(-3) x 1 = -3

(-3) x 0 = 0

(-3) x (-1) = 3

(-3) x (-2) = 6

It shows that the product of a negative number by a negative number is


positive.

To summarize, the rules in multiplying integers are:

1. The product of two integers with the same signs is positive.


(+)x(+)=+ ; (-)x(-)=+
2. The product of two integers with different signs is negative.
(+)x(-)=- ; (-)x(+)=-

88
Division of Integers

If subtraction is the opposite operation of addition. How do we relate


division to multiplication? The relation is the same. Division is the opposite
operation of multiplication. We can solve a multiplication problem through
division.

Example:
In 5 x ___ = 10, to find the missing factor let us use division. 10 which is the
product will be divided by 5, the given factor. It will come up with a result of 2.
To check, 5 x 2 = 10.

Another example: 4 x ( -3 ) = -12


Therefore, (-12) ÷ 4 = -3
(-12) ÷ (-3) = 4

The rules in multiplication of integers is the same in division.

1. The quotient of two integers with the same signs is positive.


(+)÷(+)=+ ; (-)÷(-)=+
2. The quotient of two integers with different signs is negative.
(+)÷(-)=- ; (-)÷(+)=-

Let’s Work Together

A. Find the product of each of the following:


1. (-6) (-2) 6. (-5) (-1)
2. (8) (-10) 7. (20) (4)
3. (-3) (12) 8. (-11) (-4)
4. (-4) (-6) 9. (-9) (5)
5. (-7) (-7) 10. (-8) (-7)
B. Give the quotient of each of the following:
1. 8 ÷ 8 6. 36 ÷ (-9)
2. 16 ÷ (-2) 7. 144 ÷ (-12)
3. 44 ÷ 11 8. 27 ÷ (-3)
4. (-63) ÷ (-7) 9. (-81) ÷ (-9)
5. 121 ÷ (-11) 10. 24 ÷ (-4

89
Do On Your Own

A. Give the value of x.


1. (5) ( x) = -45 6. (-65) ÷ 5 = x
2. (-13) (-12) = x 7. (-66) ÷ x = 11
3. (x) (-8) = 48 8. x ÷ (-8) = 24
4. (-11) (-2) = x 9. (-125) ÷ x = 25
5. (12) (x) = 24 10. 81 ÷ 9 = x

B. Simplify.
1. 7 ( 5-3)
2. -8 (20 + 2)
3. -2 ( 6 + 1 – 6 + 4)
4. -9 ( 1 – 8)
5. 5 [ 6 + 2 ( 5 – 6) + 8 ( 9 – 2 )]
6. 5 + 2 ( 14 ÷7)
7. [21 ÷ (-3)] – 45
8. 16 – 4 [ 5 – 6 ÷ 2]
9. 7 (24 ÷ 6) + 5 (4 – 2)
10. 2 [ -3 + 6 (-5 + 6)] ÷ (-3)

Let’s Analyze
Solve each of the following problems.

1. Nanette can type 80 words per minute. How many words can she type
in 30 minutes?
2. How much will Noel get from changing his 425 dollars if a dollar is
equivalent to 45.00 pesos?
3. Find the average of the first five counting numbers.
4. A dozen of eggs cost 60.00 pesos. How much does an egg cost?
5. I am thinking of a number which when divided by 8, the answer is -12.
What is the number?

90
Lesson
5.4 Exponents and Exponential Notation

Name:
Level: Section: Date:
Title of the Activity:
Learning Competency: Performs multiplication and division of integers
Code: M6NS-IIh 47.3, M6NS-IIh 47.4

In the previous lesson, one learned the operations on integers.


This time, we will describe an exponent and how to express
numbers in exponential notation.

Objectives:

Describe the exponent and the base in a number expressed in


exponential notation.
Writing numbers from standard form to exponential notation and vice
versa.
Give the value of numbers expressed in exponential notation.

Can we write the expression 3 x 3 x 3 x 3 in a shorter way?

3 x 3 x 3 x 3 can be written in exponential notation 34 where 4 is the


exponent and 3 is the base.

What is an exponent? What is a base?

Exponent is a small number written at the upper-right hand of another


number called the base. It indicates the number of times the base is used as
a factor.
Base is a repeated factor.

91
Example 1: Write each of the following expressions in exponential notation.
a. 5 x 5 x 5 x 5
b. (-4) x (-4) x (-4)
c. 7 x 7 x 7 x 7 x 7
Answer:
a. 54
b. (-4)3
c. 75
Example 2: Identify the base and exponent in each expression.
a. 92 b. (-87) c. 105 d. -128
Answer:
a. Base = 9 , Exponent = 2
b. Base = -8 , Exponent = 7
c. Base = 10 , Exponent = 5
d. Base = 12 , Exponent = 8

Example 3: Evaluate the following expressions.


a. 32 + 23
b. (-4)2 + (-5)2
c. 62 – 72
d. -26 ÷ 22
Solution:
a. 32 + 23 = ( 3 x 3 ) + ( 2 x 2 x 2 )
=9+8
= 17
b. (-4)2 + (-5)2 = [(-4) x (-4)] + [(-5) x (-5)]
= 16 + 25
= 41
c. 6 – 7 = ( 6 x 6 ) – ( 7 x 7 )
2 2

= 36 – 49
= -13
6 2
d. -2 ÷ 2 = - (2 x 2 x 2 x 2 x 2 x 2) ÷ (2 x 2)
= -64 ÷ 4
= -16

92
Let’s Work Together

A. Complete the table.

Exponential Form Base Exponent


1. 63
2. 105
3. (-3)2
4. 252
5. 86

B. Write the following in exponential notation.


1. (-1) (-1) (-1) (-1) (-1) (-1)
2. (-3) (-3)
3. (15)(15)(15)
4. (8)(8)(8)(8)(8)(8)
5. (9)(9)(9)(9)(9)(9)(9)(9)

Do On Your Own

A. Identify the base and exponent in each of the following expressions.


1. 87
2. 102
3. 153
4. (-26)5
5. (-1)7
B. Expand each of the following exponential notation and give its value.
1. 54
2. (-9)3
3. 64
4. 26
5. (-8)2

93
Let’s Analyze

A. Write each expression in exponential notation and evaluate.


1. Base = 4 ; Exponent = 5
2. Base = -6 ; Exponent = 3
3. Base = -2 ; Exponent = 4
4. Base = 10 ; Exponent = 2
5. Base = 1 ; Exponent = 7

B. Simplify the following expressions.

1. 82 - 72
2. 22 + 33
3. (-5)2 + 23
4. 43 ÷ 23
5. (-2)3 x 22

94
Lesson
Order of Operations
5.5

Name:
Level: Section: Date:
Title of the Activity:
Learning Competency: Interprets and explains the groupings, exponents,
Multiplication, Division, Addition, Subtraction (GEMDAS) rule
Code: M6NS-IIe-39

The knowledge in exponents and exponential notations were


needed in performing the order of operations. One will learn how
to follow rules in simplifying numerical expressions using
(GEMDAS) Grouping, Exponent, Multiplication, Division, Addition,
5. and Subtraction.

5
Objectives:

Interpret and explain the Grouping, Exponent, Multiplication, Division,


Addition, and Subtraction (GEMDAS) rule.
Perform two or more different operations on whole numbers with or
without exponents and grouping symbols.

95
How do we simplify an expression having more than one operation? Which
operation need to perform first?
We follow the GEMDAS rule:

Grouping symbols simplify expressions with parentheses, brackets,


and braces. If there are more than one grouping symbol, start from the inner
most.

Exponent evaluate the exponential term.

Multiplication perform these operations which ever comes first


Division from left to right

Addition perform these operations which ever comes first


Subtraction from left to right

Examples: Simplify the following numerical expressions.


a. 23 x ( 24 – 5 ) + 10 = 23 x 19 + 10
= 8 x 19 + 10
= 152 + 10
= 162

b. ( 45 + 55 ) ÷ 25 + 42 x 5 = 100 ÷ 25 + 42 x 5
= 100 ÷ 25 + 16 x 5
= 4 + 16 x 5
= 4 + 80
= 84

96
Let’s Work Together

A. Name the GEMDAS rule that should be done first.


1. 4+6x3
2. 43 – 6 ÷ 2
3. ( 24 + 6 ) ÷ 5 + 82
4. 28 ÷ 4 x 9 – 6
5. 5 +10 – 4

B. Write an operation symbol on each blank to make each equation true.


1. ( 5 ___ 4 ) ___ 6 = 26
2. ( 6 ___ 2 ) ___ 4 + 33 = 29
3. 63 ___ 7 ___ 52 ___ 10 = 24
4. 48 ___ ( 23 ___ 24 ) = 16
5. 12 ___ 3 ___ 9 = 4

Do On Your Own

Simplify each expression.


1. 54 ÷ 6 + ( 23 – 5 )
2. 8 + ( 6 x 3 ) + ( 30 ÷ 5 ) x 42
3. 30 – ( 4 x 6 ) ÷ 3 + 10
4. 52 – ( 20 + 4 ) ÷ 6 + ( 10 + 5 )
5. 34 – [ 2 + ( 25 – 5 ) ] ÷11

Let’s Analyze

Answer the following:

1. Which operation will you perform first in the expression 10+56÷8x3 –2?

2. What is the value of 63 + ( -2)4 – 32?

3. Evaluate: ( 4 + 6 )2 ÷ 25 + 8 x 3 + ( 63 ÷ 7 ) – 52

97
Lesson
Solid Figures
6.1

Name: COLLECTING DATA


Level: Section: Date:
Title of the Activity:
Learning Competency: Visualizes, describes and differentiates solid figures from
plane figures
Code: M6GE-IIIa-49

In this lesson, one will learn how to visualize, describe, and


differentiate solid figures from plane figures; to illustrate and
identify the different solid figures using various concrete and
pictorial models and to visualize and describe the different solid
figures.

Objectives:

Visualize, describe, and differentiate solid figures from plane figures


(M6GE-IIIa-27-28)
Illustrate and identify the different solid figures using various concrete and
pictorial models (M6GE-IIIb-29-30)
Visualize and describe the different solid figures: cube, prism, pyramid,
cylinder, cone, and sphere (M6GE-IIIc-31)

We encounter infinite figures or objects in our daily life, but some people don’t
realize that these figures have classifications.

Inside the box are two different objects, a piece of paper and glass jar. Imagine
how they look like in real life.

1. 2.

98
Based on your imagination, how many dimensions does a piece of paper have?
How about the glass jar? How many dimensions does it have?
A piece of paper is an example of a two- dimensional (2D) object which is called
plane figure while the glass jar is a three- dimensional (3D) object called solid
figure or sometimes called space/spatial figure.

Identify whether the figures inside the box is a plane or a solid figure.

Given are the different quadrilaterals. If you are going to connect the given
quadrilaterals, what figure can you form?

99
The possible figure,

This is how it is assembled,

What object resembles the formed figure?

Rectangular box or rectangular prism is a solid figure. A solid figure is a


three -dimensional (3D) figure made up of plane figures where all sides are
joined together to form a closed figure. It has length, width and height or
depth. Solid figures are also called spatial or space figures.

Two of the solid figures are the prism and pyramid, while others are cube,
cone, cylinder, and sphere. Prism and pyramid are polyhedrons for their
faces are flat surfaces called polygonal regions. The cone, cylinder and
sphere are not polyhedrons for their faces are not flat surfaces; therefore,
they are not polyhedrons.

100
Polyhedron is made up of polygons which are joined at their edges (the
intersection of two faces). Vertex is a point where edges meet.

Dimensions:

height

width

length

Parts: vertex

edge

face

Let us check the rectangular prism.

a. How many faces does it have? ( 6)

b. How many edges does it have? ( 12)

c. How many vertices does it have? (8)

The two parallel faces (top and bottom) are the bases, and the remaining faces
(front and back, left and right sides) are the lateral faces.

101
Prism versus Pyramid
Prism Pyramid

Square Prism (Cube) Square Pyramid

The prism has two polygonal The pyramid has one polygonal
bases (top and bottom) base (bottom)
The lateral faces are quadrilaterals. The lateral faces are triangles.

The name of the prism and pyramid depends on the shape of their bottom base.
It is true to both prism and pyramid that the number of the edges of their bottom
bases is equal to the number of their lateral faces.

How to determine the number of vertices, edges, bases and lateral faces?

Lateral Faces
Polyhedrons Vertices Edges Bases
(lf)
Prism sx2 sx3 2 s = lf
( top and bottom)
1
Pyramid s+1 sx2 s = lf
(bottom)

Note: s = the number of sides of the bottom base of the polyhedron

102
Illustration:
Figure 1: Pentagonal Prism

Vertices = s x 2 Formula
= 5x2 substitute the value of
s which is 5.
= 10

Edges =sx3 Formula


=5x3 substitute the values of
s which is 5
= 15
Bases = 2 Top and bottom
Lateral Faces (lf) =s
=5

Figure 2: Pentagonal Pyramid

Vertices = s + 1 Formula
= 5+1 substitute the value of
s which is 5
= 6

Edges = sx2 Formula


= 5x2 substitute the value of
s which is 5
= 10

Base =1 Bottom

103
Lateral Faces (lf) = s
= 5

Some common prisms and pyramids.

Prisms Pyramids

Triangular Prism
Triangular Pyramid

Note:
If all the faces of a pyramid are
regular triangles it is called
tetrahedron.

Rectangular Prism Rectangular Pyramid

104
Hexagonal Prism
Hexagonal Pyramid

Classify the figures below whether they are prism or pyramid.

These are the non-polyhedrons.

Cone Cylinder Sphere

105
Do they have vertices? Only the cone has a vertex.

Do they have flat surfaces? The cone has a circular flat surface and it is its
bottom base while the cylinder has two circular flat surfaces, the top and
bottom bases.

Sphere is a round solid figure with every point on each surface with
equidistant from its center.

Let’s Work Together

A. Draw a if the statement is true and if it is false.

_______ 1. A cylinder does not have parallel faces.

_______ 2. A cone has three vertices.

_______ 3. An ice cream cone represents a pyramid.

________4. Prisms and pyramids are polyhedrons.

________ 5. The common endpoint where three edges meet is called the
vertex.

B. Identify each solid figure. Give the name of solid, vertices, edges, and
lateral faces.

Figures Name of Vertices Edges Lateral


Solids Faces

106
Do On Your Own

Draw the top, front, and bottom view of the following solid figures.
Figure Top Bottom Front

1.

2.

3.

107
4.

5.

Let’s Analyze

Identify what is being described in each number. Write your answer


on the line provided before the number.

_________ 1. A solid figure that has two flat faces and can roll

_________ 2. A solid figure without any flat faces and can roll

_________ 3. A solid figure that is like a prism, but has circular bases

_________ 4. A polyhedron with at least a pair of parallel faces

__________ 5. A polyhedron with only one base

108
Lesson Nets of Solid
6.2
Name:
Level: Section: Date:
Title of the Activity:
Learning Competency: Identifies the nets of the following space figures: cube, prism,
pyramid, cylinder, cone and sphere using plane figures.
Code: M6GE-IIIb-54

In this lesson, one will learn to identify the nets of the following
space figures: cube, prism, pyramid, cylinder, cone, and sphere,
using plane figures and create models of different solid figures
such as circle, cube, prism, pyramid, cylinder, cone, and sphere
using plane figures.

Objectives:

Identify the nets of the following space figures: cube, prism, pyramid,
cylinder, cone, and sphere using plane figures (M6GE-IIIc-32)
Create models of different solid figures such as circle, cube, prism,
pyramid, cylinder, cone, and sphere using plane figures

What are the objects below?

If we are going to unfold the objects, how will they look like?

109
Choose the letter of the correct answer at the right.

How does a shoe box look like if you unfold it? ____ A.

A tent? ____

An ice cream cone?_____

B.

C.

When a solid figure is unfolded, it forms a flat or two-dimensional surface


that follows a pattern and it is called a net. Solid figures may have different
nets. Nets are useful in finding the surface area of the solids.

Determine whether a net forms a solid figure:

1. See to it that the solid figure and the net have the same number of faces
and that the shapes of the faces of the solid figure match the shapes of
the corresponding faces in the net.

2. Figure out how the net is to be folded to form the solid figure and see to
it that all the sides fit together properly.

110
Try to figure out the nets of the following solid figures by matching column A
to column B.

A B

1. A.

Sphere

2. B.

Cone

3. C.

Cylinder

4. D.

Triangular Pyramid
5.

E.

6. Rectangular Prism F.

111
Construct nets of the following figures:

1.

Rectangular Pyramid

2.

Cylinder

Let’s Work Together

Box the objects or solid figures that are prisms.

1. 4.

2. 5.

112
3.

Do on Your Own

Identify the solid figures of the following objects. Write your answer on the
space provided.

1. 2.

_______________ _______________

3. 4.

_______________ _______________

113
5. 6.

_______________ _______________

7. 8.

_______________ _______________

5. 10.

_______________ _______________

114
Let’s Analyze

Name the solid figure that can be formed by folding each pattern. Write your
answer on the space provided before the number.

_____ 1. ------- 6.

_____ 2. ____ 7.

_____ 3. ____ 8.

_____ 4. ____ 9.

_____ 5. ____ 10.

115
Lesson Writing Rules for Sequences
7.1

Name:
Level: Section: Date:
Title of the Activity:
Learning Competency: Formulates the rule in finding the nth term using different
strategies
Code: M6AL-IIIb-55.0

In this lesson, one will learn how to formulate the rule in finding
missing term in the sequence by using different strategies.

Objectives:
COLLECTING DATA
Formulate the rule in finding the nth term using different strategies
(looking for a pattern and working backward) (M6AL-IIId-7)

Patterns are everywhere. They are repeated designs or recurring sequences.


Finding and understanding patterns will help make educated guesses and
discover mathematical relationship.

In Mathematics, a sequence is a set that consists of numbers or objects that


usually follow a particular pattern. Each number in the sequence is called a
term or sometimes an element or a member.

In the given sequence below,

1, 3, 5, 7, 9

116
1 is the first term (a1), 3 is the second term (a2), 5 is the third term (a3), and so
on. The nth term of a sequence is written as an. Since the sequence ended up
in a certain number which is 9 in our example then it is called a finite.
Another example of a finite sequence:

, , ,

1 , 2 , 3 , 4

The first term of the sequence is 1 and the last is 4. Since it has a last term the
sequence is finite.
Take a look at the next sequence:
2, 4, 6, 8, 10,…

The first term is 2, the second term is 4, and so on. The ellipsis (…) shows
that an established pattern continues, meaning it has no end. Hence, the
sequence is called an infinite sequence.
Example.

, , , , ...

1, 3, 5, 7, …

117
The first term of the sequence is 1, the second term is 3, the third term is 5, the
fourth term is 7 and the following terms are still to be determined as indicated
by the “…”.
More examples of sequences.

An increasing sequence is one in which every term is greater than the


previous term. That is an+1 ˃ an.

The following sequences are both increasing.

3, 5, 7, 9, 11

5, 10, 15, 20, 25, …

A decreasing sequence is one in which every term is less than the previous
term. That is an+1 ˂ an.

The following sequences are both decreasing.

5, 4, 3, 2, 1

15, 12, 9, 6, 3, …

Formulate the rule in finding the nth term for each sequence using counting
numbers (all positive numbers such as 1, 2, 3, 4, …).

2 , 4 , 6 , 8 , 10 , ___ , ____ , …

The missing terms are the nth terms


d - common difference (second term – first term, third term -second
term, and so on)

4–2=2 you notice that the difference are the same which is 2
6–4=2 and that is called common difference (d)

118
8-6=2
10 – 8 = 2
d=2

To find a missing number, first find a rule behind the sequence. Sometimes by
just looking at the numbers, we can already determine the pattern.

Rule: an = 2n
an nth term or the missing term
n term number

The missing 6th and 7th terms are 12 and 14.

Where a6 = 2n Rule
a6 = 2(6) substitute 6 to n
a6 = 12 6th term

for the 7th term:


a7 = 2n Rule
a7 = 2 (7) substitute 7 to n
a7 = 14 7th term

5 , 4, 3, 2, 1 (finite sequence)

4 – 5 = -1
3 – 4 = -1
2 – 3 = -1
1 – 2 = -1
d = -1

Row 1: Term Number: 1 2 3 4 5


Row 2: -1 -2 -3 -4 -5
Row 3: 5 4 3 2 1

119
Row 1 is composed of term numbers. To find Row 2, the given sequence
is decreasing by 1 each time and that gives us the common difference
(d) of -1. Multiplying -1 to the terms in Row 1 to get the terms in Row 2.
Such as,

d x Term number = Term in Row 2

-1 x 1 = -1
-1 x 2 = -2
-1 x 3 = -3
-1 x 4 = -4
-1 x 5 = -5

Now, try to work out how you get from the terms on the 2 nd
row to the terms on the 3rd row. To find the terms in Row 3 is by adding 6 to
the terms in Row 2.

Term in Row 2 + 6 = Term in Row 3


-1 + 6 = 5
-2 + 6 = 4
-3 + 6 = 3
-4 + 6 = 2
-5 + 6 = 1

Therefore, the rule is

Rule: an = -1n + 6

Apply the rule to get the terms in the given sequence.


a1 = -1n + 6
= -1(1) + 6 substitute the term number 1 to n
= -1 + 6
= 5 First term (a1)

a2 = -1n + 6
= -1(2) + 6 substitute the term number 2 to n
= -2 + 6
= 4 Second term (a2)

120
a3 = -1n + 6
= -1(3) + 6 substitute the term number 3 to n
= -3 + 6
= 3 Third term (a3)

a4 = -1n + 6
= -1(4) + 6 substitute the term number 4 to n
= -4 + 6
= 2 Fourth term (a2)

a5 = -1n + 6
= -1(5) + 6 substitute the term number 5 to n
= -5 + 6
= 1 Fifth term (a2)

Study the given sequence. Find the missing term/s.

1) 3, 8, 13, 18, 23, ____, _____ …

5 5 5 5

d = 5.

Term Term Pattern Rule Rule nth Rule


Number
1 3 5(1) -2=3 Multiply the
2 8 5(2)-2=8 common
3 13 5(3)-2=13 difference (5) 5n-2
4 18 5(4)-2=18 by the term
5 23 5(5)-2=23 number then
6 28 5(6)-2=28 subtract 2.
7 33 5(7)-2=33

121
Therefore, the 6th and the 7th terms are 28 and 33 respectively.

Using the same sequence let us formulate the rule in finding the nth term
using the backward strategy.

3, 8, 13, 18, 23, _____, _____,…

5 5 5 5

d= 5

Here, we start from the last term instead of the first term.

Term Term Pattern Rule Rule nth Rule


Number
5 23 5(5) -2= 23 Multiply the
common
4 18 5(4)-2=18 difference (5) 5n-2
3 13 5(3)-2=13 by the term
then subtract
2 8 5(2)-2=8
2.
1 3 5(1)-2=3

Therefore, the 6th and the 7th terms are 28 and 33 respectively.

122
Let’s Work Together

Give the next three terms in the following patterns:

1. 15, 18, 21, 24, 27, ____,_____,_____, …

2. 88, 86, 84, 82, 80, ____,_____,_____, …

3. 8, 10, 12,14,16, ______, _____,_____,…

4. 41, 45,49,53,57,______,______,_____, …

5. 30, 28, 26,24,22, _____,______,_____, …

6. 12, 24, 36, 48, 60, ____,_____,______, …

7. 5,10, 15, 20, 25, ______,_______,_____, …

8. 1, 6, 11, 16, 21, 26, ____,____,______, …

9. 12,16,20,24,28,______,______,______, …

10. 21, 20,19,18,17,_____,_____,_____, …

Do on Your Own

Write the rule in finding the nth term.


Sequence Rule

1. 9, 12, 15, …

2. 12, 9, 6, …

3. 10, 15, 20,25,…

123
4. 8, 12, 16, 20, …

5. 8, 6, 4, 2,…

Let’s Analyze

Analyze the certain patterns of the given sequences then show the rules and
the next three terms.

Sequences Rule nth Term Rule Next two terms

1.
8, 13, 18,…
2.
16, 12, 8 ,…
3.
17, 18,19,…
4.
6, 11, 16,…
5.
20, 17, 14,…

124
Lesson Algebraic Expression
7.2

Name:
Level: Section: Date:
Title of the Activity:
Learning Competency: Gives the translation of the real life verbal expressions and
equations into variables
Code: M6AL-IIIC-57

In this lesson, one will learn how to define a variable in an


algebraic expression, give the translation of real-life verbal
expressions and equations into letters or symbols and vice versa,
and represent quantities in real-life situation using algebraic
expressions and equations.

Objectives:

Define a variable in an algebraic expression (M6AL-IIIe-17)


Give the translation of real-life verbal expressions into letters or
symbols and vice versa (M6AL-IIIe-16)
Represent quantities in real-life situations using algebraic
expressions ( M6AL-IIIe-18)

Lesson 2.1 Variable and Algebraic Expression

Algebraic Expression is a mathematical phrase involving variables, numbers


and one or more operations.

In an algebraic expression, any letter or symbol that represents a number is


called a variable. In this expression, a number may replace a variable. A
specific number represents a constant. A term is a number or variable, or a
product or quotient of a number and variables raised to a specified power. The
terms are separated by plus (+) or minus (-) sign. It is not necessary to write

125
the + sign at the beginning of a term. The numerical factor of a term is called
numerical coefficient while the letter factor is called literal coefficient.

Study the following examples:

Given an algebraic expression 4m2 + 25

How many terms are there? 2 ( 4m 2 , 25)

What is the variable? m

What is the numerical coefficient? 4

What is the literal coefficient? m2

What is the constant term? 25

Let’s try another example.

Given an algebraic expression a3 - b3 + c3

Number of terms 3 (a3, -b3, c3)

Variables a, b and c

Numerical Coefficient 1 for a 3 = 1a3

-1 for -b3 = -1b3

1 for c3 = 1c3

Literal Coefficient a3, -b3, c3

Constant term none

126
Let’s Work Together

Decide whether the following is a constant or a variable.


________1. The number of months ending with “ber” in a year
________ 2. The water consumption in each month
________ 3. The number of hours in a day
________ 4. The number of patients in the hospital each day
________ 5. The number of pupils in a particular school each year
________ 6. The number of days in a week
________ 7. The Meralco bill each month
________ 8. The number of months in a year
________ 9. The tuition fee of a college student in a particular school
________ 10. The value of π (pi) in finding the area of a circle

Do on Your Own

Encircle the variable/s and box the constant term in each expression.
1. 20 + m

2. 12( x – y)

3. 100mn

4. 11s

5. 2s – 5

6. 3x2+ 3y

7. 15b – 2c + 7

8. 4x + y - 25

9. r3+ 3r2 -r

10. y

127
Let’s Analyze

A. Identify the number of terms, variables and constant term used in the
following algebraic expressions.
Algebraic Number Variables Constant Term
Expressions of
Terms
1. -3x3 + x2 +2
2. 5mn4
3. 8y-10
4. 5(y + z)
5. πr2
𝟑𝒃𝒅
6.
𝟒

7. 4x2- 5y
8. abc
9. -14x2y + 3
10. 4c + 8d – e

B. Identify the numerical and literal coefficients of each expression.


Algebraic Numerical Literal Coefficient
Expression Coefficient
1. 29bc
2. 8d2- 4d+2
3. t2+ 36
4. 16h – 9m
5. 12pq + 27p

128
Lesson 2.2 Give the translation of real-life verbal expressions into letters
or symbols and vice versa.

Why do we need to translate English phrases into mathematical phrases?


Before we answer the foregoing question, let us first know the meaning of
words and phrases used in mathematical problems.

To translate word phrases into algebraic expressions, the important thing to do


is to familiarize the words and phrases associated with symbols or operations.

Mathematical Symbols Words/phrases


Operations/Symbols
Addition + increased by, more than, the sum of,
plus, added to, the total of,
Subtraction - minus, less, less than, the difference of,
decreased by, diminished by, subtracted
from, taken away from
Multiplication x , . , () times, of, the product of, twice, multiplied
by,
Division ÷, — Divided by, the quotient of, the ratio of,
Equality symbol = is, equals, is the same as, is equal to

Here are some of the different word phrases can be translated or written as
algebraic expressions.

Word Phrases Algebraic Expression

4 added to z z +4
4 more than z

4 less than z z - 4
4 subtracted from z
4 taken away from z

129
Let’s try these.

Translate the following phrases to algebraic expressions


a. eleven subtracted from n
b. the quotient of x and 25
c. the sum of y and negative three
d. twice k
e. the difference of a number and twelve divided by four

Solutions:
a. eleven subtracted from n
We are looking for the difference of 11 and n. Which comes first? Why? n
comes first followed by 11 because of the phrase, subtracted from.
The answer is: n – 11

b. the quotient of x and 25


You are looking for the quotient of x and 25. It is very obvious that quotient
means division, so, as x is your dividend and 25 is your divisor.
𝒙
The final answer is:
𝟐𝟓

c. the sum of y and negative three


Our key word here is the sum which means addition. So, we get the sum of
six and negative three. Take note that one of our term is negative and that
is translated to negative sign (-). You are not allowed to omit – sign. So, we
have y + (-3).
The final answer is: y + (-3)

d. twice k
The keyword twice means multiply by 2. We translate this as 2k

e. the difference of a number and twelve divided by four

130
Look at the expression and examine it closely. What have you observed?
Did you see that the expression contains an unknown? What shall we do?
We will use a variable to represent an unknown by letting y be the number
or y = the number.

The difference of a number and twelve divided by four.

Translated to: y – 12 ÷ 4

𝒚−𝟏𝟐
Therefore, the result is:
𝟒

Let’s Work Together

Match the phrases in column I with their equivalent algebraic expressions in


column II. Write your answer on the blank provided before the number. Use y
for the variable.

I II
______1. thrice w subtracted from one hundred A. 4y + 10 .
______ 2. twice a number minus five B. y – 11
______ 3. increase the product of a number and 4 by 10 C. 10+y
______ 4. a number increased by fifty D. 2y – 5
𝟐𝟓
______ 5. y diminished by eleven E.
𝒏
______ 6. the quotient of twenty-five and n F. 100 – 3w
______ 7. a number times twelve G. 20 + 2y
______8. thrice the sum of a number and eight H. 12y
______9. ten increased by a number I. y + 50
______10. twenty plus twice a number J. 3(y+8)

131
Do on Your Own

Complete the chart below by writing the corresponding word


phrases for each algebraic expression.

No. Algebraic Word phrases


Expressions
1 2a -7
𝑛
2
2
3 3x - 2

4 -8x + 5

5 2y + 5
12
6 −8
𝑚
7 3x + 12

8 2mn

9 21b -2

10 15 + d

132
Let’s Analyze

Write an algebraic expression to answer each question.


1. Let M be nanay ‘s age now.
a. What was her age 10 years ago?
b. What was her age 12 months ago?
c. What will be her age after 10 years?

2. Let Z be the number.


a. What is thrice the number?
b. What is the difference of the number and 12?
c. What is the square of the number?

Lesson 3.3 Represent quantities in real-life situations using algebraic


expressions

The skill that we learned from the previous lessons help us to represent
quantities in real-life situations.

Read and analyze the problems.


1. Jude weighed n kilograms. Express in algebraic expression his weight after
he gained 2 kilograms.
Solution:
The variable n is the unknown weight of Jude while the word gained implies
addition. Therefore, we can express it as n + 2.
2. Sissy is x years old.
a. Represent her age 4 years from now
b. Represent her age 4 years ago

133
Solution:
x is Sissy’s present unknown age
a. The phrase from now implies addition; hence, the expression will be
x + 4.
b. The word ago implies subtraction; hence, the expression will be x - 4 .

Let’s Work Together

Translate the following word phrases into algebraic equations.


Write your answer on the line.

1. Twenty subtracted from five times k__________________________

2. Twice the difference between eighty-eight and z________________

3. Seven times y plus two times x _____________________________

4. The ratio of s to p ______________________________________

5. The square of n ______________________________________

Do on Your Own

Write the word phrases for each algebraic expression.

1. Prt =

2. 4b – 12 =

3. 5x + n =

4. 12cz =

5. 15 + 3n =

134
6. 100k =

7. 8 ( k + 6) =

8. N +2 =

9. N3 =

10. LW =

Let’s Analyze

A. Translate the word phrases into algebraic equations.


1. The sum of fifty and twice the number
2. Twelve times a number divided by four
3. The sum of twice five and negative three

B. Write an algebraic expression to answer each question.


1. Let W be the time now.
a. What was the time six hours ago?
b. What is the time after eight hours?

135
Lesson Algebraic Equations
7.3

Name:
Level: Section: Date:
Title of the Activity:
Learning Competency: Gives the translation of the real life verbal expressions and
equations into variables
Code: M6AL-IIIC-57

In this lesson, one will learn how to define a variable in an algebraic


equation, give the translation of real-life verbal equation into letters
or symbols and vice versa and represent quantities in real-life
situations using algebraic equations.

Objectives:

Define a variable in an algebraic equation (M6AL-IIIe-17)


Give the translation of real-life verbal equation into letters or symbols
and vice versa (M6AL-IIIe-16)
Represent quantities in real-life situations using algebraic equations
( M6AL-IIIe-18)

Problems:

1. Yñigo’s height is x centimeters (cm). Express in


algebraic expression his height after he gained 10 cm.

x is Yñigo’s unknown present height, while the word


gained means addition. Hence, we express this as:
x + 10

2. Yñigo’s height is x centimeters (cm). If he becomes 10


cm taller next year, his height will be 40 cm.

136
In this problem, x is Yñigo’s unknown present height. The phrase becomes
taller implies addition; 40 cm will be his total height. Therefore, we express
this as: x + 10 = 40.

Let’s us compare the answers in Problems 1 and 2.


Answer number 1 is x + 10 while answer number 2 is x + 10 = 40.

What have you observed between the two answers?


What do you call x in the expression/equation?

x is a variable (any letter or symbol that represents a number)


Which answer gives you a complete information?

x + 10 = 40 gives you a complete information because of the relation


symbol (=) and 40 as the result while x + 10 does not give a complete
information for it has no answer and no relation symbol and we call this as an
algebraic expression.

x + 10 = 40 is an algebraic equation. An algebraic equation is a mathematical


statement stating that two algebraic expressions are equal. The equality symbol
(=) indicates that the expression on the left side of an equation is equal to the
right.

Given: 2x + 5 = 13

2x +5 and 13 are both expressions and they are equal.

1. Antonio, when asked about his age, replies, “ I am four-years younger than my
sister.” Express his age in algebraic equation if his age now is 32.

To solve , let x be the age his sister.

Then, translate:

four years younger than my sister is 32

x -4 = 32

Therefore, the algebraic equation for Antonio’s age is x - 4 = 32

137
Let’s Work Together

Write the following statement as equations.

1. A number increased by 10 is equal to 23. Find the number.

2. Eighty -two decreased by a number is -24. Find the number.

3. The sum of a number and -42 is 40. Find the number.

4. What number diminished by 43 is -50?

5. What number increased by 29 is 34?

6. Four times a number is added to the number, giving 45. Find the number.

7. When 15 is added to five times a number, the result is 20. Find the number.

8. The sum of a number and five is multiplied by 2 , giving 26 as a result . Find

the number.

9. 31 less than a number is 79. Find the number.

10. The sum of two integers is 52. One of the integers is 13. What is the other

integer?

Do On Your Own

A. Let n be the unknown number. Translate each word sentence into an

algebraic equation.

1. The product of 2 and a number is 18.

2. The difference of a number and 5 is 20.

3. Nine minus four times a number is 5.

4. The square of a number is 81.

5. One more than a number is 100.

138
Let’s Analyze

Write a word sentence for each equation.

1. N+ 12 = 24

2. 8n – 6 = 2n
𝑛
3. 3 = 3

4. x2 + 1 = 10

5. 4n = 24

6. 20 – n = 20

7. 5n -7 = 28

8. 2b + 4 = 6

9. 3 + 2m = 36

10. 2n + 2 = 4

139
Lesson
7.3
Solving Equations

Name:
Level: Section: Date:
Title of the Activity:
Learning Competency: Solves routine and non-routine problems involving different
types of numerical expressions and equations.
Code: M6AL-IIId-60.0

In this lesson, one will learn how to solve and create routine and
non-routine problems involving different types of numerical
expressions.

Objectives :

Solve routine and non-routine problems involving different types of


numerical expressions and equations (M6AL-IIIf-19)
Create routine and non-routine problems involving numerical
expressions and equations ( M6AL-IIIf-20)

Word Problems:

1. A basket of fruits has atis and guava. The atis are 4 times the number of
guava. There are 20 fruits in all. How many of each kind of fruits are there
in the basket?

Let’s analyze and solve the given problem.

Let n be the number of guavas.

a. What is asked?
The number of each kind of fruits in the basket

140
b. What are the given facts?
 n guavas
 4n atis
 20 fruits
c. What operation is to be used?
Addition
Division
d. What is the number sentence?
4n + n = 20

e. Solution

4n + n = 20 ( equation)
solve for the value of n:

4n + n = 20

5n = 20 combining like terms

5𝑛 20
= dividing both sides by the numerical coefficient
5 5
which is 5 to get the value of n

n = 4 Solution of an equation ( number that makes


the sentence true)
( 4 is the number of guavas)
If n = 4 then, 4n = 4(4) = 16 ( number of atis)
Therefore, the number of guavas is 4 while the number of atis is 16.

To check:
4n + n = 20 (equation)
4 (4) + (4) = 20 substitute the value of n which is 4.

16 + 4 = 20
20 = 20 True

141
Therefore, the solution is correct.

2. Angel sold half of her magazines and then


bought 10 more. She now has 22. How
many did she begin with?

Let n be the original number of magazines.


a. What is asked?
The number of magazines she had at first

b. What are given?


𝒏
 magazines
𝟐

 10 magazines
 22 magazines

c. What operation is to be used?


Addition
Subtraction
Multiplication

d. What is the number sentence?

𝒏
+ 10 = 22
𝟐

142
Solution:

𝒏
+ 10 = 22 We need to remove +10 by
𝟐
subtracting

𝒏
+ 10 - 10 = 22 – 10 subtracting both sides by 10
𝟐

𝒏 1
(𝟐) ( ) = (𝟏𝟐) (𝟐) then, we need to remove 2 so that
𝟐
the remaining term on the left side
is the variable n by multiplying
both sides by 2.
n = 24 the original number of magazines

Therefore, Angel had 24 pcs of magazine at first.

To check:
`
𝒏
+ 10 = 22 substitute the value of n which is 24
𝟐

24
+ 10 = 22
2

22 = 22 True

Therefore, the solution is correct..

143
The first problem is an example of routine problem while the second is a non-
routine problem. Routine problem solving involves using at least one of the
four fundamental arithmetic operations and/or ratio to solve problems that are
practical in nature. A non-routine problem is any complex problem that
requires some degree of creativity or originality to solve. They typically do not
have an immediately apparent strategy for solving them. These problems can
be solved in multiple ways.

Create a Word Problem using the given data to show the present weight.
1. Given:

Dom is x kilogram in weight.


x – 2 kg
29 kg a month ago

Possible Problem:

Dom is x kilogram (kg) in weight. A month ago, he was lighter by 2 kg.


and he weighed 29 kg. How heavy is he now?

How to Create a Word Problem

1. Determine the scenario the problem wants you to create.


2. Analyze the given information and expression.
3. Determine the operations to be used.
4. Construct the equation.
5. Check whether the equation formed meets the required scenario or
situation.

144
Let’s Work Together

Translate each sentence into an algebraic equation. Write your answer on

the line.

1. A number increased by six is fifteen. _______________

2. Ten times a number is fifty. _______________

3. If four times a number is added to nine the result is forty-nine. ________

4. The sum of a number and five is thirty. _______________

5. Three times a number increased by ten is forty. _______________

6. Four more than twice a number is twenty. _______________

7. The sum of a number and six is twelve. _______________

8. Twice a number decreased by two is six. _______________

9. The ratio of a number to 10 is 3. __________________

10. Fifty divided by twice a number is equal to two. __________________

Do on Your Own

Write a simplified algebraic equation for each problem then solve.

1. Nico weighs 20 kilograms (kg) plus half of his own weight. How heavy does

Nico weigh?

2. There are 32 pupils in the school canteen. The boys are 4 more than the

girls. How many boys are there?

145
3. Pam saved Php 200.00 this week from her allowance. If this amount is

Php50.00 more than the amount she saved last week, how much did she

save last week?

4. The sum of three consecutive numbers is 63. Find the numbers.

Let’s Analyze

Solve each of the following problems.

1. Carmela brought a basket of 28 fruits for her grandmother. The basket

contains mangoes and papayas. The mangoes are six times more than the

papayas. How many of each kind of fruits are there?

2. Nimfa, Angel, and Mary were all candidates for president in their Math Club.

Ninfa got 16 more votes than Angel, who got twice as many votes as Mary.

There were 76 votes counted in all. How many votes did each candidate

get?

146
Lesson Speed, Distance and Time
8.1
Name:
Level: Section: Date:
Title of the Activity:
Learning Competency: Calculates speed, distance and time.
Code: M6GE-lIIe-62

In this lesson, one will learn to apply knowledge of speed in


mathematical problems and real-life situations.

Objectives:

Calculates speed, distance, and time. (M6ME-IIIg-17)


Solves word problems involving speed. (M6ME-IIIg-18)

Speed refers to how fast an object moves. An object is said to be in


uniform motion if it moves in a straight line without changing its speed. The
examples that follow show the relationship between speed, distance and time.

Example 1: A 100-meter dash between student A and student B happened


today. It took student A 45 seconds to cover the distance, while
student B covered it in 60 seconds.

a. What is the speed of student A?


To calculate for the speed, divide the distance travelled
𝑑𝑖𝑠𝑡𝑎𝑛𝑐𝑒 𝑡𝑟𝑎𝑣𝑒𝑙𝑙𝑒𝑑 (𝑑)
over time, that is, 𝑒𝑑 (𝑠) = .
𝑡𝑖𝑚𝑒 (𝑡)
𝑑 100 𝑚𝑒𝑡𝑒𝑟𝑠
𝑠= = = 2.22 𝑚/𝑠 (Note: m/s is read as
𝑡 45 𝑠𝑒𝑐𝑜𝑛𝑑𝑠
“meters per second”)
The speed of student A is 2.22 m/s.
b. Can you compute for the speed of student B?

c. What can you say about the speed of the two students

147
Example 2: A car travelling a distance of 150 kilometers arrived at its
destination after 3 hours. What is the speed of the car?

𝑑 150 𝑘𝑖𝑙𝑜𝑚𝑒𝑡𝑒𝑟𝑠
Solution: 𝑠 = = = 50 𝑘𝑚/ℎ
𝑡 3 ℎ𝑜𝑢𝑟𝑠
The speed of the car is 50 km/h.

Try this!
An athlete walks 20 kilometers in 4 hours. What is the athlete’s speed?

Example 3: The cheetah is one of the fastest animals on earth. It has a speed
of 61 miles per hour. What is the distance it covered after running for 5 hours?

Solution: To calculate the distance travelled, just multiply the speed and time.
𝑑𝑖𝑠𝑡𝑎𝑛𝑐𝑒 𝑡𝑟𝑎𝑣𝑒𝑙𝑙𝑒𝑑 = 𝑠𝑝𝑒𝑒𝑑 𝑥 𝑡𝑖𝑚𝑒
61𝑚𝑖
𝑑 = 𝑠𝑡 = ( ) (5ℎ) = 305 𝑚𝑖

The distance travelled by the cheetah in 5
hours is 305 miles.

Example 4: How much distance is covered in 7 hours with a speed of 55 km


per hour?

55 𝑘𝑚
Solution: 𝑑 = 𝑠𝑡 = ( ) (7 ℎ) = 385 𝑘𝑚

The distance travelled in 7 hours is 385 kilometers.

Try this!
Roman travels at a speed of 22 meters per minute. If he travels for 10
minutes, find the distance covered by Roman.

Example 5: How much time will be taken to cover a distance of 400 km with a
speed of 70 km per hour?

Solution: To calculate the time taken, divide the distance


travelled by the speed.
𝑑𝑖𝑠𝑡𝑎𝑛𝑐𝑒 𝑡𝑟𝑎𝑣𝑒𝑙𝑙𝑒𝑑
𝑡𝑖𝑚𝑒 = = 400 𝑘𝑚 ÷ 70 𝑘𝑚/ℎ = 5.7 ℎ
𝑠𝑝𝑒𝑒𝑑

148
Try this!
Victoria walks 280,000 meters, as part of her every day routine, at a
rate of 70,000 m/h. How much time did she take to travel the distance?

The time taken to cover the distance of 400 km is


approximately 5.7 hours.

Example 6: A runner covers a distance of 18 kilometers at a speed of 9 km


per hour. Compute for the time taken to cover the distance.

𝑑
Solution: 𝑡 = = 18 𝑘𝑚 ÷ 9 𝑘𝑚/ℎ = 2 ℎ
𝑠
The time taken to cover the distance of 18 km is 2
hours.

Real-life situations may also involve solving for the speed, distance, and
time. Here are some examples:

1. Ms. Virgie is going to Santo Tomas, Batangas from Parañaque City. She
rides an airplane that has a speed of 900 km/h. The distance between
these two places is 47 km. On her return home, she travelled via plane
that has a speed of 500 km/h. How much time did it take for her way
back? (Express your answer in minutes.)

Solution: From Santo Tomas, Batangas to Parañaque:


𝑑 𝑘𝑚
𝑡= = 47 𝑘𝑚 ÷ 500 = 0.09 ℎ
𝑠 ℎ
𝑚𝑖𝑛𝑢𝑡𝑒𝑠
0.09 ℎ 𝑥 60 ℎ𝑜𝑢𝑟
= 5.4 𝑚𝑖𝑛𝑢𝑡𝑒𝑠
It took Ms. Virgie 5.4 minutes to travel back home.

2. Leo traveled 260 km in the first 4 hours. For the next 6 hours, he travelled
420 km. What is his average speed for the whole journey?

𝑡𝑜𝑡𝑎𝑙 𝑑𝑖𝑠𝑡𝑎𝑛𝑐𝑒 𝑐𝑜𝑣𝑒𝑟𝑒𝑑


Solution: 𝐴𝑣𝑒𝑟𝑎𝑔𝑒 𝑠𝑝𝑒𝑒𝑑 = 𝑡𝑜𝑡𝑎𝑙 𝑡𝑖𝑚𝑒 𝑡𝑎𝑘𝑒𝑛
260+420 𝑘𝑚 680 𝑘𝑚
= = = 68 𝑘𝑚/𝑠
4+6 𝑠 10 𝑠
The average speed of Leo is 68 km/s.

Try this!
Elizabeth left school and went to her friend’s house at a speed of 50
km/h. After an hour, Francis left school
149 and went to the opposite
direction at a rate of 60 km/h. At what time will they be 600 km apart?
Let’s Work Together

Answer the following.

1. A van travels 400 km in 8 hours. What is the average


speed of the car?

2. A storm is moving toward your house at a speed of 30


km/hr. It is now 90 km away from your house. In what
time will the storm reach your house?

3. What is the speed of a jet plane that flies 8000 km in 10


hours?

4. Ronald can swim at a speed of 3.9 m/s for 7 minutes.


What distance will she cover in that time?

5. At recess you got mad and threw your sandwich at a


speed of 5 m/s. It flies for 20 seconds through the air.
How much distance did the burrito cover?

Do on Your Own

Solve the following.


1. It takes Bry 0.5 hours to drive to school. Her route is 27 km long. What is
Bry’s average speed on her drive to school?

2. While playing, a boy threw a flying saucer 240 meters through the air.
While in the air, the saucer traveled at an average speed of 24.0 m/s.
How long did the disk remain in the air?

150
3. How far can your friend get if he can travel at 3.6 m/s and in 6 seconds you
will discover that his squirt gun has run out of paint?

4. You need to get to class, 300 meters away, and you can only walk in the
corridors at about 1.5 m/s. (running in the corridors are not allowed ).
How much time will it take you to get to your class?

6. A plane travels 406,000 meters in 1000 seconds. What was its speed?

Let’s Analyze

1. Bill and Amy want to ride their bikes from their neighborhood to school
which is 25.5 kilometers away. It takes Amy 50 minutes to arrive at
school. Bill arrives 30 minutes after Amy. How much faster (in
meters/second) is Amy’s average speed for the entire trip?

2. Two cars started from the same point, at 6 am, traveling in opposite
directions at 50 and 60 mph respectively. At what time will they be 560
miles apart?

3. At 10 am a car began a journey from a point, traveling at 50 mph. At 11 am


a jeep started traveling from the same point at 70 mph in the same
direction as the car. At what time will car B pass car A?

151
Lesson Area
8.2
Name:
Level: Section: Date:
Title of the Activity:
Learning Competency: Find the area of plane figures.
Code: M6GE-lIIe-62

In this lesson, one will learn to apply knowledge and understanding


of area of figures.

Objectives:

Find the area of plane figures. (M6ME-IIIh-89)


Solve word problems involving area of plane figures. (M6ME-IIIh-90)

Time to Learn

The area of any plane figure refers to the number of square units
contained within the figure. Areas of polygons can be calculated by applying
the formulas indicated below.

Rectangle: 𝐴𝑟𝑒𝑎 (𝐴) = 𝑙 × 𝑤


Square: 𝐴𝑟𝑒𝑎 = 𝑠 2
Parallelogram: 𝐴𝑟𝑒𝑎 = 𝑏 × ℎ
1
Triangle: 𝐴𝑟𝑒𝑎 = 2 × 𝑏 × ℎ
1
Rhombus: 𝐴𝑟𝑒𝑎 = 2 × 𝑑1 × 𝑑2
1
Trapezoid: 𝐴𝑟𝑒𝑎 = 2 ℎ(𝑏1 + 𝑏2 )
Circle: 𝐴𝑟𝑒𝑎 = 𝜋𝑟 2

152
Example 1: Find the area of a parallelogram whose base is 24 cm and whose
height is 7 cm.

Solution: 𝐴𝑟𝑒𝑎 = 𝑏 × ℎ
= 24 × 7
𝐴 = 168 𝑠𝑞𝑢𝑎𝑟𝑒 𝑐𝑒𝑛𝑡𝑖𝑚𝑒𝑡𝑒𝑟

Example 2: Find the area of a circle with a radius of 30 inches.

Solution: 𝐴𝑟𝑒𝑎 = 𝜋𝑟 2 (Use 𝜋 = 3.14)


= 3.14 × 302
= 3.14 × 900
𝐴 = 2,826 𝑠𝑞𝑢𝑎𝑟𝑒 𝑖𝑛𝑐ℎ𝑒𝑠

Try this!
Find the area of a triangle whose base is 5 centimeters and whose
height is 8 meters.

Let’s Work Together

Find the area of each shape.

1. 2.

153
3. 4.

5.

Do on Your Own

Solve each problem

1. If the height of a parallelogram is 22 m and the base is seven m, what is


the area of the parallelogram?

2. Rita created a triangular shaped garden and needs to put down fertilizer to
cover the space. If the garden has a base of 2.5 m and a height of 3.7
m, how much fertilizer will she need?

3. Camille bought a triangular shaped carton with a height of seven inches


and a base of three inches. She plans to cover the front with a piece of
fabric, how much fabric will she need?

4. Nicole wants to make a screen saver for his new computer. The screen is
43 inches long by 29 inches wide. What is the area of the screen?

5. Katherine is buying a cover for her side table. If the table is six feet on all
sides, what is the area of the cover?

154
Let’s Analyze

Solve each problem.

1. Maki is painting one wall in her bedroom. The wall measures 24 feet long
and 10 feet wide. If one can of paint cover 60 square feet will it be
enough? Explain.

2. Carlo has brownies. The length of each brownie is eight cm and the width
is five cm. Find the area of the brownies.

3. How many square centimeters of colored paper will cover one face of the
cardboard if its dimensions are 40 cm by 28 cm?

4. Which has a smaller area: a square with a side of 12 cm or a circle with a


diameter of 10 cm? How much smaller?

5. Mang Kardo’s goat is tied to a tree with a


nine meter rope as shown in the picture.
Around how many square meters of
grazing area does the goat have?

155
Lesson Surface Area
8.3
Name:
Level: Section: Date:
Title of the Activity:
Learning Competency: Visualizes and describes surface area and names the unit of
measure used for measuring the surface area of solid/space figures.
Code: M6GE-lIIe-67

In this lesson, one will learn to apply knowledge and understanding


of surface area of plane and solid figures.

Objectives:

Visualize and describe surface area and use the appropriate unit of
measure. (M6ME-IIIi-91)
Find the surface area of cubes, prisms, pyramids, cylinders, cones and
spheres. (M6ME-IIIi-93)
Solve word problems involving measurement of surface area. (M6ME-
IIIi-94)

You have already learned that a plane figure has two dimensions and
these are length and width. Some examples of plane figures are square and
rectangle. A space figure, or three-dimensional figure, is a solid figure which
has three dimensions -- length, width, and height. Examples of this are prism,
pyramid, cylinder, cone, and sphere.

A rectangular solid has 6 plane regions,


each of which is a rectangle and are called faces.
A cube is a rectangular solid, of which all
faces are squares.

156
A prism is a solid figure with two
identical ends called the bases and flat
sides. The prism is named by the shape
of its base.

A pyramid is a solid figure


where the sides are triangles which
meet at the top (apex) and the
base is any polygon. The pyramid is
named by the shape of the base.

A cylinder is a solid figure which has two circular (or elliptical) identical
flat ends and one curved face.
A sphere is a solid figure which
has a surface that is always the same distance from its center.
A cone is a solid figure that has a
circular base and one vertex.

Surface area of these types of figures is the sum of the areas of its outer
surface. It refers to the number of square units that can be contained on the
surface of the solid. To calculate the surface area of some solids, we have the
following formula:

Cube: 𝑆𝑢𝑟𝑓𝑎𝑐𝑒 𝐴𝑟𝑒𝑎 (𝑆𝐴) = 6𝑠 2 , where 𝑠 = 𝑠𝑖𝑑𝑒


Prism: 𝑆𝑢𝑟𝑓𝑎𝑐𝑒 𝐴𝑟𝑒𝑎 (𝑆𝐴) = 2(𝑙𝑤 + 𝑙ℎ + ℎ𝑤), where 𝑙 = 𝑙𝑒𝑛𝑔𝑡ℎ,
𝑤 = 𝑤𝑖𝑑𝑡ℎ and ℎ = ℎ𝑒𝑖𝑔ℎ𝑡
Cylinder: 𝑆𝑢𝑟𝑓𝑎𝑐𝑒 𝐴𝑟𝑒𝑎 𝑆𝐴 = 2𝜋𝑟 2 + 2𝜋𝑟ℎ, where 𝑟 = 𝑟𝑎𝑑𝑖𝑢𝑠 and
( )
ℎ = ℎ𝑒𝑖𝑔ℎ𝑡
2
Cone: 𝑆𝑢𝑟𝑓𝑎𝑐𝑒 𝐴𝑟𝑒𝑎 (𝑆𝐴) = 𝜋𝑟 + 𝜋𝑟𝑠, where 𝑟 = 𝑟𝑎𝑑𝑖𝑢𝑠 and
𝑠 = 𝑠𝑙𝑎𝑛𝑡 ℎ𝑒𝑖𝑔ℎ𝑡
2
Pyramid: 𝑆𝑢𝑟𝑓𝑎𝑐𝑒 𝐴𝑟𝑒𝑎 (𝑆𝐴) = 𝑏 + 2𝑏𝑠, where 𝑏 = 𝑏𝑎𝑠𝑒 and
𝑠 = 𝑠𝑙𝑎𝑛𝑡 ℎ𝑒𝑖𝑔ℎ𝑡
Sphere: 𝑆𝑢𝑟𝑓𝑎𝑐𝑒 𝐴𝑟𝑒𝑎 (𝑆𝐴) = 4𝜋𝑟 2, where 𝑟 = 𝑟𝑎𝑑𝑖𝑢𝑠

157
Example:

Finding the surface area of solid figures can also be applied in real-life
situations.

Example:

1. May is making an accessories box in the shape


of a rectangular prism. The box will have the
dimensions shown at the right. The cost of
painting the exterior of the box is ₱26 per
square inches. How much does May have to
spend to paint the jewelry box ?

Solution: To find the total cost, you need to find first the surface area of
the box.
𝑆𝑢𝑟𝑓𝑎𝑐𝑒 𝐴𝑟𝑒𝑎 (𝑆𝐴) = 2(𝑙𝑤 + 𝑙ℎ + ℎ𝑤)
= 2(26 × 23 + 26 × 7 + 23 × 7)
= 2(598 + 182 + 161)
𝑆𝐴 = 1 882 𝑠𝑞𝑢𝑎𝑟𝑒 𝑖𝑛𝑐ℎ𝑒𝑠 (𝑖𝑛2 )
𝑇𝑜𝑡𝑎𝑙 𝑐𝑜𝑠𝑡 = 𝑆𝐴 × 𝑐𝑜𝑠𝑡 𝑝𝑒𝑟 𝑠𝑞𝑢𝑎𝑟𝑒 𝑖𝑛𝑐ℎ
= 1 882 𝑖𝑛2 × ₱26
𝑇𝑜𝑡𝑎𝑙 𝑐𝑜𝑠𝑡 = ₱48,932
Answer: May has to spend ₱48,932 to paint the box.

2. Find the surface area of the earth assuming the earth to be a sphere which

158
has a radius of 6,371 km.

Solution: 𝑆𝑢𝑟𝑓𝑎𝑐𝑒 𝐴𝑟𝑒𝑎 (𝑆𝐴) = 4𝜋𝑟 2 (Use 𝜋 = 3.14)


= 4 × 3.14 × 63712
= 4 × 3.14 × 40,589,641
𝑆𝐴 = 509 805 890.96 𝑘𝑚2
Answer: The surface area of the earth is 509 805 890.96 km2.

Try this!
Find the surface area of a cube with sides of 5 inches

Let’s Work Together

1. 2.

3. 4.

5. 6.

159
Do on Your Own

Solve each problem.

1. Paulo’s bedroom measures 6 m by 5 m by 4.6 m. How many square


meters of wallpaper are needed to cover the four walls?
2. How many square centimeters of wrapping paper is needed to cover a
shoe box which measure is 36 cm by 26 cm by 30 cm?
3. A cylindrical can which has a radius of 6 cm and height of 26 cm must be
made from aluminum. How much aluminum is needed?
4. An ice cream cone has a radius of 5 cm and a slant height of 22 cm. Find
its surface area.
5. Buboy wants to paint a styro ball with a radius of 25 cm. What is the
surface area of the ball that he has to paint?

Let’s Analyze

Solve each problem.

1. A basket shaped like a rectangular prism is made of wood. The base is a


30-cm square and the height is 40 cm. How many square centimeters
of wood did it take to make the basket?

2. How much cloth border is needed to cover a box that is 23 cm by 9 cm by 4


cm?

3. What is the surface area of a cube whose edge measures 7 meters?

4. A wooden box that is in the shape of a rectangular prism has the following
2
dimensions: the length is 10 inches, width is 3 inches, and height is 2 3
inches. Find the total cost of silver coating for the entire box if the cost
per square inch is ₱30.
5. Three-fourths of the earth's surface is covered with water. Find the surface
area covered by water if the earth can be assumed as a sphere with a
radius of 6,371 km

160
Lesson Volume of Solid Figures
8.4
Name:
Level: Section: Date:
Title of the Activity:
Learning Competency: Determine the relationship of the volume between: a
rectangular prism and a pyramid, a cylinder and a cone, and a cylinder and a sphere.
Code: M6ME-IVe-70

In this lesson, one will learn to apply knowledge and understanding


of volumes of solid figures.

Objectives:

Find the volume of cylinders, pyramids, cones and spheres. (M6ME-IVb-


97)
Solve word problems involving volume of solid figures. (M6ME-IVc-98)

Volume refers to the amount of space an object occupies. It is measured


in cubic units. Examples of solid figures are:

When a prism and a pyramid have the same height and the same base,
the volume of the prism is greater than the pyramid. If a cone and a cylinder
have bases with equal areas, and both have the same height, then the volume

161
2
of the cone is one-third the volume of the cylinder. The volume of a sphere is 3
the volume of a cylinder with same radius and height equal to the diameter.

The volume of a prism is found by multiplying its base and height. The
volume of a rectangular prism is the product of its length, width, and height.
Example 1: JR has a chocolate box whose
length is 15 cm, height 12 cm,
and width 9 cm. Find the
volume of the box.

Solution: The given box has length = 15 cm,


width = 9 cm, and height = 12 cm.
𝑉𝑜𝑙𝑢𝑚𝑒 = 𝑙𝑒𝑛𝑔𝑡ℎ × 𝑤𝑖𝑑𝑡ℎ × ℎ𝑒𝑖𝑔ℎ𝑡
= 15 × 9 × 12
= 1 620 𝑐𝑢𝑏𝑖𝑐 𝑐𝑚

Example 2: The base of a right prism is a right triangle with legs 18 and 24
cm. If the volume of the prism is 648 cu cm, find the height of
the prism.

1
Solution: 𝐴𝑟𝑒𝑎 𝑜𝑓 𝑡ℎ𝑒 𝑏𝑎𝑠𝑒 = 2 × 18 × 24
= 216 𝑠𝑞𝑢𝑎𝑟𝑒 𝑐𝑚
𝑉𝑜𝑙𝑢𝑚𝑒 = 648 𝑐𝑢 𝑐𝑚
𝑉 648
𝐻𝑒𝑖𝑔ℎ𝑡 𝑜𝑓 𝑡𝑟𝑖𝑎𝑛𝑔𝑢𝑙𝑎𝑟 𝑝𝑟𝑖𝑠𝑚 = 𝐴 = 216
= 3 𝑐𝑚

The volume of a cylinder can also be found by applying the same


formula used in finding the volume of a prism.
Example 3: Calculate the volume of a cylinder where the radius of the base is
28 cm and the height is 20 cm.

Solution: 𝑉𝑜𝑙𝑢𝑚𝑒 = 𝐴𝑟𝑒𝑎 𝑜𝑓 𝑡ℎ𝑒 𝑏𝑎𝑠𝑒 × ℎ𝑒𝑖𝑔ℎ𝑡


= 𝜋𝑟 2 ℎ (Use 𝜋 = 3.14)
2
= 3.14 × 28 × 20
𝑉𝑜𝑙𝑢𝑚𝑒 = 49 235.2 𝑐𝑢𝑏𝑖𝑐 𝑐𝑒𝑛𝑡𝑖𝑚𝑒𝑡𝑒𝑟

The volume of a pyramid is one-third the product of the area of the base
and the height.

162
Example 4: Find the volume of a pyramid with a rectangular base measuring
24 m by 16 m and height 40 m.

Solution:
1
𝑉𝑜𝑙𝑢𝑚𝑒 = 3 × 𝑎𝑟𝑒𝑎 𝑜𝑓 𝑡ℎ𝑒 𝑏𝑎𝑠𝑒 × ℎ𝑒𝑖𝑔ℎ𝑡
1
= 3 × (24 × 16) × 40
𝑉𝑜𝑙𝑢𝑚𝑒 = 5 120 𝑐𝑢𝑏𝑖𝑐 𝑚𝑒𝑡𝑒𝑟

1
The volume of a cone is equal to the product of the area of the base
3
times the height.

Example 5: Calculate the volume of a cone if the


height is 24 in and the radius is 14 in.

1
Solution: 𝑉𝑜𝑙𝑢𝑚𝑒 = 3 × 𝑎𝑟𝑒𝑎 𝑜𝑓 𝑡ℎ𝑒 𝑏𝑎𝑠𝑒 × ℎ𝑒𝑖𝑔ℎ𝑡
1
= 3 × 𝜋 × 𝑟2 × ℎ
1
= 3 × 3.14 × 142 × 24
1
= 3 × 3.14 × 142 × 24
𝑉𝑜𝑙𝑢𝑚𝑒 = 4,923.52 𝑐𝑢𝑏𝑖𝑐 𝑖𝑛𝑐ℎ𝑒𝑠

2 4
The volume of a sphere is the volume of a cylinder or getting the
3 3
product of pi and the cube of the radius.

Example 6: Calculate the volume of sphere with radius 8 cm.

4
Solution: 𝑉𝑜𝑙𝑢𝑚𝑒 = 3 𝜋𝑟 3
4
= 3 × 3.14 × 83
4
= 3 × 3.14 × 512
𝑉𝑜𝑙𝑢𝑚𝑒 = 2 143.57 cubic centimeter

163
Try this!
1. A shoe box is 2.70 m long, 0.7 m wide and 0.6 m high. How
much soil will the box hold if it is filled to the top?

2. Find the volume of a can with base radius of 7 cm and height


of 20cm.

3. Find the volume of a square pyramid whose base measures


23 cm by 23 cm and whose height is 29 cm.

4. Find the volume of a cone having r = 37 cm and h = 60 cm.

5. Find the volume of a sphere whose radius is 9 cm.

Let’s Work Together

Find the volume of each solid.

1. 2.

3. 4.

164
5. 6.

Do on Your Own

Solve each problem

1. What would be the length of Apple’s box if she wants to fill it with 75,000
m3 of sand? The width of the box is 90 m and its height is 43 m.

2. Alexis has marbles with diameter measuring 2.6 cm each. Around how
many such marbles could fill up a container with a seven cm radius
and a 20 cm height?

3. What is the volume of a regular cylinder whose base has radius of 160 mm
and has height of eight cm? (Express your answer in cm.)

4. An aquarium is 20.6 inches wide, 86 inches long and 49 inches high. What
is its volume?

5. Mang Arsenio is a balloon vendor. He prepares 15 balloons for a kiddie


party by blowing air in it. If a balloon has a circumference of 47 cm
when blown, how many cubic centimeters of air does all 15 balloons
contain?

165
Let’s Analyze

Solve each problem.

1. What is the volume of a regular cylinder whose base has radius of 28 cm


and has height of 22 cm?

2. A cubical box has dimensions 20in × 7in × 20in. How many cubes with
dimension 4 in x 5 in x 6 in can be placed inside the box?

3. Pineapple juice is sold in aluminum cans that measure 26 inches in height


and 25 inches in diameter. How many cubic inches of pineapple juice
are contained in a full can?

4. A cylindrical bottle is 20 cm deep and 27 cm wide. How much liquid can the
glass hold?

5. Find the volume of the regular rectangular pyramid with a rectangular base
that has a length of 24 meter and a width of 29 meter, and a height of
26 meter.

166
Lesson Meter Reading
8.5
Name:
Level: Section: Date:
Title of the Activity:
Learning Competency: Read and interprets electric and water meter readings.
Solves routine and non-routine problems involving electric and water consumption.
Code: M6ME-IVc-75.0, M6ME-IVc-76.0

In this lesson, one will learn to apply knowledge and understanding


of electrical and water readings in mathematical problems and real-
life situations.

Objectives:

Solve word problems involving electric and water consumption.


Create problems involving electric and water consumption, with
reasonable answers.

Have you seen an electric and water meter? The following are samples
of electric and water meters. These objects are used to measure how much
water or electricity is consumed for a certain period of time. The picture on the
right is the water meter, while the other two pictures show an analog and digital
electric meter, respectively.

167
Meter readings are done every month. How do we read water and electric
meters and compute the consumption? Here are the steps:
a. For water meter
Step 1: Read the meter and record the number
shown.
Step 2: To calculate the consumption for the
month, subtract the current reading from the
previous reading. The difference would be the
amount of water consumed.

b. For electric meter


Step 1: Read the meter from left to
right. Notice that the dials move in
opposite directions.
Step 2: Look at the numbers
that the pointers have passed in the dials one at
a time and write down the numbers in order. The
reading on the meter on the right is 8416.
Step 3: To calculate the electric consumption for the month,
subtract the previous reading from the current reading. The
difference is the number of kilowatt hours consumed.

Study the following situations.

Example 1: The electric meter reading last month was 8732 kWh. The
present reading is 9458 kWh. How much electricity was
consumed in one month?

Solution: 9458 − 8732 = 𝑛


9458
− 8732
726 kWh was consumed in one month.
726

Example 2: The current water reading at Sol’s diner is 4313 m3. The previous
reading was 4235 m3. How much water was consumed for the
month?

Solution: 4313 − 4235 = 𝑛

168
4313
− 4235
78 m3 was consumed for the month.
78

Example 3: At the end of March, the electric meter reads 4095. At the end of
April, the meter reads 5932. At the end of May, it showed 6731.
How much electricity is consumed in April than in May?

Solution: March to April  5932 − 4095 = 1837 𝑘𝑊ℎ


April to May  6731 − 5932 = 799 𝑘𝑊ℎ
1837 − 799 = 𝑛
1837
− 799
1038 kWh was consumed in April than in May.
1038

Try this!
1. Last June, our electric meter reads 8096 kWh. This month, our
reading is 9439 kWh. If electricity costs ₱10.4067 per kWh, how
much do we need to pay?

2. Last month, the water reading by Maynilad was 2453 m 3. For


the following month, the water reading is 2406 m3. If water costs
₱33.60 per cubic meter, how much will Patrick pay at the Bayad
Center?

Let’s Work Together

A. Write the meter reading for each item on the blank provided.
1.
____ ____ ____ ____

2.
____ ____ ____ ____

169
3.
____ ____ ____ ____

4.
____ ____ ____ ____

5.
____ ____ ____ ____

B. Complete the water meter reading.


Previous Reading Present Reading
Water Consumption (in m3)
(in m3) (in m3)
78 2 210
2 254 2 342
2 425 2 564
2 711 2 867
2 909 2 016

Do on Your Own

Complete the table below.

A. Water Consumption
Amount to
Current Previous Consumption Pay (₱33.60
Month
(in m3) (in m3) (in m3) per cubic
meter)
January 3 067 3 654
February 2 838 2 039
March 3 165 46
April 6 432 58
May 2 156 92

170
B. Electric Consumption
Amount to
Current Previous Consumption Pay
Month
(in kWh) (in kWh) (in kWh) (₱10.4067
per kWh)
June 5 510 5 349
July 6 310 6 257
August 1 486 304
September 9 164 457
October 3 210 694

Let’s Analyze

Solve each problem.

1. Frances records the electric reading every 15 days. On November 30, the
reading was 5 936. How much electricity was used after 15 days if the
reading was 6 185?

2. An ordinary LED lamp uses 60 watts an hour. How many kilowatt will
twenty LED lamps consume in September if they are used for 8 hours
every night?

3. How much electricity is consumed in 5 months by a 140-watt double door


refrigerator if the cost per kilowatt hour is ₱10.41?

4. Ofelia’s household uses water for their everyday activities. She kept a
record of their consumption for 3 months as follows:
Initial Reading 97 023 m3
October (1st reading) 213 639 m3
November (2nd reading) 220 798 m3
December (3rd reading) 243 150 m3
What is their average daily consumption?

5. A swimming pool is 29 m long, 21 m wide, and 2.86 m deep. How much will
it cost to fill the pool with water, if a cubic meter costs ₱33.60?

171
Lesson Collecting Data
9.1
Name:
Level: Section: Date:
Title of the Activity:
Learning Competency: Collecting data in one or two variables
Code: M6ME-IVc-78

In this lesson, one will understand data gathering and creating f


frequency tables.

Objectives:

Collect data on one or two variables using any source. (M6SP-IVe-1.6)

Data are facts or information about ideas, objects or events. There are
different ways on how you can collect data: interview, questionnaires,
observations, experiments, or surveys. After collecting the data, one way of
organizing it is presenting it in table form. Records of the number of Netflix
movies Olivia watched from March to June is shown below as an example.

Months March April May June


Number of Netflix movies 9 7 11 11

Try this!
Make a table for the sizes of Coke ordered from Mang Oliver’s Resto.
The following are the results (S for Mismo, M for 500L, L for 1.5L): L,
M, S, S, L, M, L, S, L, L, L, L, S, L, L, L, L, L, L, S, S, M, S, S, L, L, M,
L, M, S, S, L, S, L, M, S, S, M, M, L

172
Another way of organizing data is by constructing a frequency
distribution table. A tally mark is used to record how often a particular score or
number occurs. The number of times a score or number appears is called the
frequency.
To construct a frequency distribution table:
a. List the scores or numbers from highest to lowest (or lowest to
highest).
b. Use tally marks to record how often each score or number appears.
c. Count the marks and record it in the frequency column.

Example: Construct a frequency distribution table for this set of data.


There are 20 students who took a scholarship exam consisting of
50 questions and the results are as follows:
35 33 28 34 30 35 35 35 28 29
34 27 27 31 31 35 30 34 32 36

Scores in a Scholarship Examination


Scores Tally Frequency
36 l 1
35 llll 5
34 lll 3
33 l 1
32 l 1
31 ll 2
30 ll 2
29 l 1
28 ll 2
27 ll 2
Total n = 20 n = 20
(The result shows that there are more students who got a score of 30 and above
than those who got below 30.)

The tables and frequency distributions will help us interpret and analyze
data easier.

Try this!
Make a frequency distribution table of the last digit of each student’s
LRN (Learner’s Reference Number): 4, 0, 2, 3, 0, 1, 2, 5, 7, 6, 5, 6, 3,
7, 3, 8, 1, 0, 4, 5, 6, 2, 9, 9, 2, 8, 5, 6, 9, 4, 4, 6, 4, 0, 2

173
Let’s Work Together

Use the frequency table to answer the questions that follows:

Students of Rainbow Elementary School were asked about their favorite


color and the results are shown below:

Favorite Color Tally Frequency


Black llll l
Green llll llll llll lll
White l
Purple llll l
Yellow llll
Red ll
Blue llll llll llll llll llll llll llll llll
Orange llll llll llll llll lll

1. Complete the frequency column.


2. What color was the most favorite?
3. What color was the least favorite?
4. How many students like green?
5. How many students like purple?
6. How many students like red?
7. How many students like orange?
8. How many students like black more than white?
9. How many students like blue more than yellow?
10. How many students were surveyed?

174
Do on Your Own

Make a survey of the birth month of your classmates. Create a frequency


distribution table and answer the questions that follow.

Birth Month Tally Frequency


January
February
March
April
May
June
July
August
September
October
November
December

1. How many of your classmates have March as their birth month?

2. How many of your classmates have August as their birth month?

3. How many of your classmates have June as their birth month?

4. In what month are the most number of your classmates born?

5. In what month are the least number of your classmates born?

6. How many students took part in the survey?

175
Let’s Analyze

Students’ scores in a 40-item Numeracy Skill Test are listed below:


30 33 22 30 26 28 20 30 33 29 27
23 36 31 26 21 27 27 33 27 32 32
26 27 31 33 28 27 25 30 27 27 32

1. Arrange the scores from highest to lowest.


2. Make a frequency distribution table for the given data.
3. What is the lowest score?
4. What is the highest score?
5. How many students scored below 30?
6. How many students got a score of 30 and above?
7. What score occurred most often?
8. What interpretation can you state from the frequency distribution
table?

176
Lesson Pie Graph
9.2
Name:
Level: Section: Date:
Title of the Activity:
Learning Competency: Construct a pie graph based on a given set of data, Interprets
data presented in pie graphs.
Code: M6ME-IVd-79.0, M6ME-IVd-80.0

In this lesson, one will learn how to construct and interpret pie graphs.

Objectives :
Construct a pie graph based on a given set of data. (M6SP-IVe-2.6)
Interprets data presented in a pie graph. (M6SP-IVf-3.6)

A pie graph (or circle graph) shows how a whole is divided. It presents
data in a circular chart which is divided into sections called sectors. It also
shows the comparison of each part or sector to the whole circle.

Constructing a Pie Graph

To construct a pie graph, just follow the steps:

1. Arrange the data.


2. Convert each data into percent.
3. Compute the number of degrees for
each percent. (Note: A circle
has 360𝑜 .)
4. Construct a circle and plot the data with the use of a protractor.
5. Write the name and the corresponding percent for each sector.

177
Example: Amelia is assigned to present to class a pie graph of her allowance.
She decides to divide her daily allowance of ₱100 according to
the following:

Expense Amount
Food ₱50
Transportation 20
Savings 10
Others 20

Solution:
After organizing her data, she then converted each amount into percent
as follows:
Food: (50 ÷ 100)𝑥100 = 50%
Transportation: (20 ÷ 100)𝑥100 = 20%
Savings: (10 ÷ 100)𝑥100 = 10%
Others: (20 ÷ 100)𝑥100 = 20%
She then computed the number of degrees for each item.
Food: 0.50𝑥360 = 180𝑜
Transportation: 0.20𝑥360 = 72𝑜
Savings: 0.10𝑥360 = 36𝑜
Others: 0.20𝑥360 = 72𝑜
Using a protractor, Amelia then constructed her pie graph by dividing the
circle into sectors corresponding to each item.

178
Try this!
During its annual fair, Kalayaan Elementary School was able to raise
₱16 500. Create a pie graph showing the given data: Plant Sale ₱1
155, Games and Rides ₱7 425, Arts and Crafts Sale ₱2 145,
Rummage sale ₱2 475, and Food and Drinks Stand ₱3 300.

Interpreting a Pie Graph

Remember that information in a given data corresponds to each section


in a pie graph. The smaller the percent of the data, the smaller is its part in the
circle. Likewise, the bigger the percent of the data, the larger is its part in the
circle.
There are 1 500 enrollees in Masagana Elementary School. This pie
graph shows the enrollment for each grade level. Use the graph to answer the
following questions.

1. How many pupils are enrolled in Grade V?


1
2. Which of the grade levels have an enrollment that comprise of the
4
circle?
3. What is the total enrollment for Grades III and IV?
4. What grade level has an enrollment which is higher than Grade VI?

179
5. What is the total enrollment for Grades I to VI?
6. What grade levels has the same number of enrollees?
7. How much more is the total enrollment in Grades IV - VI than in
Grades I - III?

Try this!
Construct a pie graph showing the number of each flavor of ice
candy that Harry sold on Saturday then answer the questions that
follow: Chocolate - 45 Avocado - 20
Strawberry - 25 Grapes - 10
a. Which flavor of ice candy is most bought by the customers?
b. What percent of the grapes ice candy were sold that day?
c. How many more strawberry ice candy than avocado was
bought that day?

Let’s Work Together

In the class of Teacher Isabella, 50 students were asked to indicate the


place that they would like to visit in the Philippines during the summer vacation.
The table below shows the result:

Places of Interest Number of Students


Bohol 9
Palawan 20
Davao 15
Cebu 2
Sorsogon 4

Draw a pie graph to illustrate the results.

180
Do on Your Own

Create a survey to find out the number of siblings that your classmates
have. Record your results using the table below

Number of Siblings Number of Students


0
1
2
3
4
More than 4

Using the data that you have gathered, draw a pie graph and write 3
questions based on your graph.

Let’s Analyze

In the election of the Math Club president in a certain elementary school,


there were 200 members who voted. These were the results:

Candidate Percent of Votes Received


Isla 35%
Emily 15%
Jack 50%

1. Find the number of votes for each candidate.


2. Construct a pie graph and make an appropriate title for the graph.

181
Lesson Word Problems
9.3
Name:
Level: Section: Date:
Title of the Activity:
Learning Competency: Creates problem that can be answered using information
presented in a pie graph.
Code: M6ME-IVe-82

In this lesson, one will learn how to solve and create word
problems from information presented in a pie graph.

Objectives:

Solves routine and non-routine problems using data presented in a pie


graph. (M6SP-IVf-4.6)
Creates problems that can be answered using information presented in
a pie graph. (M6SP-IVf-6)

A pie graph, also known as circle graph, divides a whole circle into
sectors which represents the information in a given data. Suppose you are
given a diagram showing the four barangays that participated in a certain
contest. How will you answer the following questions?

182
a. How many families took part in the contest?
b. What is the percentage of the families that joined in each barangay?

Solution:
a. There are 100 families who joined in Barangay Matulungin
There are 225 families who joined in Barangay Maganda
There are 50 families who joined in Barangay Masipag
There are 125 families who joined in Barangay Maginhawa
There are 500 families who took part in the contest.

b. Barangay Matulungin = 20%


Barangay Maganda = 45%
Barangay Masipag = 10%
Barangay Maginhawa = 25%

Try this!
The graph shows the budget for a year of the Santos
family with a yearly income of ₱120 000.

1. What percent is budgeted for


a. food?
b. transportation and school allowances?
c. rent, electricity, and water?
d. others?
2. How much do they plan to spend on
a. food?
b. transportation and school allowances?
c. rent, electricity, and water?
d. others?

183
Let’s Work Together

Work in groups:

1. Write down 10 different songs. Make a survey in your class about


their favorite song in the list (or at least what they would be listening to
the most) and find out the number of students that liked each item.
Show the information that you have gathered in table form.

2. Draw a pie graph to show that information that you have gathered.

3. Use the diagram to create five (5) word problems and ask your classmates
to answer them. Check their works afterwards.

4. Present your group work to the class.

Do on Your Own

1. The pie graph below shows how much money was saved on the allowance
of 5 students in a week.

a. Who had the most savings?


b. By how many percent is Oscar’s
savings less than Harper?
c. How much less is Arlo’s savings
than Freddy’s?
d. Give suggestions on how you can
save your allowance in a week.

184
2. The pie graph below shows the number of students absent during an exam
in Makisig Elementary School.

a. What grade level has the least number of absences?

b. By how many percent of the students are absent in Grade II compared with
Grade IV?

c. How many students were absent during the exam?

d. What suggestions can you give to prevent a student from being absent
during an exam?

185
Let’s Analyze

The graph shows what percent of the city’s ₱1 000 000 community
development budget was spent on as issued by Mayor Reyes. Use the diagram
to answer the following questions.

1. How much was spent on the Cleanliness and Beautification


Projects?

2. How much was spent on the Amusement Parks, Community Library,


and Annual Cityhood Fair combined?

3. How many more percent was spent on the Youth Center than on
the Transportation System?

4. What item was given the highest budget and how much?

5. What item was given the least budget and how much?

186
Lesson Making Simple Predictions
9.4
Name:
Level: Section: Date:
Title of the Activity:
Learning Competency: Quantifies the phrases “most likely to happen”, “unlikely to
happen”.
Code: M6ME-IVf-84

In this lesson, one will learn to make a guess as to whether an event


is most likely, equally likely, or unlikely to happen.

Objectives:

Describes the meaning of probability. (M6SP-IVg-19)


Quantifies the phrases “most likely to happen", “equally likely to happen”
and "unlikely to happen." (M6SP-IVg-20)

Sometimes in your daily activities, you think of the chance of something


good or bad might happen. In this case, what you are trying to do is to get the
probability of what will happen. We can describe the event whether it will most
likely, equally likely, or unlikely to happen based on the given information.

For example, if you toss a coin , there may be two possible outcomes, either
head or tail. Outcome is the possible result in an event. Since the chance of
getting a head or tail is the same, then we can say that, they are equally likely
to happen.

187
Here is another example. If you spin this spinner, how
many possible outcomes will there be? The outcomes are
not equally likely. Why?

Now, let us have another example. Suppose the teacher


placed some coated chocolate eggs inside a jar. She asked
each of her students to pick an egg without looking inside. In
the jar are 10 green eggs, 6 yellow eggs, 4 blue eggs, and 5
red eggs. What color do you think will the students most likely
pick? Or what color will be less likely to get picked? Looking
at the number of each colored eggs, you can say that the
green eggs are most likely to be picked and the blue eggs are the less likely to
be picked. Why? What about the chance of getting a pink colored egg?

Try this!
There are 10 blue notebooks, 6 violet notebooks, and 4 black
notebooks inside the box. The teacher then asked you to get a
notebook inside the box without looking at it.
1. What is your chance of taking out a green notebook?
Why?
2. Do the notebooks have equal chances of getting picked?
Why?
3. Which notebook would you least likely to pick out? Why?
4. Which notebook would you most likely to pick out? Why?

Let’s Work Together

A. The letters of the word “MISSISSIPPI” are written on a card and placed in
a box. Jeremy picked one card without looking inside the box. Answer
the following questions.
1. What is the chance of getting an S than getting an I?

188
2. What is the least likely to be picked among
the letters?
3. What is the chance of getting a P than an
M?
4. What is the chance of getting a W?
5. How many possible outcomes are there?

B. A well-known shoe factory would like to give an


incentive to their employees who are always on time. The supervisor said that
those who are punctual can spin the wheel and may receive a prize as follows:
blue - cash incentive, red - gift certificates, violet - car, and green - free trip to
Baguio.
1. The chance of getting a blue color is __________ as the red color.
2. The chance of getting a yellow color is
__________.
3. The chance of getting a green color is
__________ as the blue color.
4. The __________ is the least likely to be
picked among the colors.
5. The chance of getting a trip abroad is
more likely than all the others. Is it
true? Why? __________

Do on Your Own

A. There are 6 black pens, 5 blue pens, and 4 red pens inside my bag. Write
the correct answer on the line.
1. What pen is most likely to be picked?
2. What pen is least likely to be picked?
3. The _____ pen is less likely to be picked than
the yellow pen.
4. The violet pen is _____ to be picked.
5. How many outcomes are there?

189
B. Mrs. Gonzaga’s Grade 6 class consists of 13 boys and 22 girls. 15 of the
students wear jackets because of the cold weather. Half of the class
are wearing their PE uniform. She wanted to pick a student to be the
class monitor.
1. What is the chance that a boy will be picked? _____
2. What is the chance that the student picked is a girl? _____
3. What is the chance that the student picked wears a PE uniform than
those who isn’t? _____
4. What is the chance that the student picked wears an
eyeglass?______
5. What is the chance that the student picked wears a jacket than the
student who isn’t? _____

Let’s Analyze

A certain beauty pageant sold out raffle tickets as part of the contest.
The committee then listed the names of the persons who are the top sellers.
Below are the names of the seller and the number of tickets that they sold.

Name of Seller Number of tickets sold


Marilen De Leon 300
Josefina Bagumbayan 200
Tomasita Asiba 800
Anthonina Cruz 300
Cesaria Bagdio 500

The tickets are placed inside the tambiolo and will be picked at random.
The ticket containing the names of the seller will receive a prize from the
sponsors.

1. How many persons are top sellers of the tickets? _____


2. Who has the least chance of getting a prize? _____
3. Who has the most chance of getting a prize? _____
4. Who among the sellers have an equal chance of getting a prize?__
5. Does Marvin Ocampo have a chance of getting a prize? _____
Why? _____

190
Lesson Probability
9.5
Name:
Level: Section: Date:
Title of the Activity:
Learning Competency: Makes simple predictions of events based on the results of
experiments.
Code: M6ME-IVg-87

In this lesson, one will learn to make a guess as to whether an


event is most likely, equally likely, or unlikely to happen.

Objectives:

Performs experiments and records outcomes.


Makes listings and diagrams of outcomes and tells the number of
favorable outcomes and chances using these listings and diagrams.
Makes simple predictions of events based on the results of experiments.
Solves word problems involving experimental and theoretical probability.
Creates problems involving experimental and theoretical probability.

Experiments and Outcomes

An experiment (or trial) is any process that can be done repeatedly and
involves chances. The set of possible results of an experiment is called an
outcome. Study the situation and identify the experiment and the outcomes.

Angela is a Math teacher. She has 5 color pens inside her bag, red,
green, violet, pink, and blue, which she uses in checking her students
notebooks. She wants the pink pen. If she picks a pen without looking, what
are her chances of getting the pink pen?

The experiment here is picking a pen. There are 5 pens, so there may
be 5 possible outcomes: red, green, violet, pink, and blue. Each of these pens
has an equal chance to be picked. Picking the pink pen is what you call a
favorable outcome.
191
Try this!
Carmen tossed a die. How many possible outcomes are there? What
are these outcomes? What is the experiment in this situation?

Outcomes: lists and tree diagrams

An event is one or more outcomes of an


experiment. An example is rolling a die and
then tossing a coin. What do you think are the
possible outcomes for these two events?

There are different ways on how you can


identify the possible outcomes of an
experiment: by listing, making a tree diagram,
or using the basic counting principle.

Example 1: How many possible answers (outcomes) are there for a Yes-No
test consisting of two questions?
a. by listing
1st Question 2nd Question
Yes Yes
Yes No
No Yes
No No
b. by making a tree diagram
Yes
1st Question

2nd Question
No
c. by basic counting principle
Notice that for each question there are 2 possible outcomes: Yes
or No. There are 2 x 2 outcomes or 4 outcomes for this event.

Example 2: Maki has just opened his new restaurant. For their menu, they have
3 different viands, 4 desserts, rice and 2 drinks. How many possible

192
combinations of a meal can he serve if a meal consists of a viand, dessert, rice
and drinks?

Solution: Using the basic counting principle:


viand x dessert x rice x drinks = meal
3 x 4 x 1 x 2 = 24 possible combinations
Can you try listing the possible outcomes?

Try this!
In an appliance store, electric fans
come in 3 styles: wall, desk, and stand
fan. They come in 6 colors: blue,
green, pink, violet, yellow, and white.
Draw a tree diagram for the following
and state the number of possible
outcomes.

Probability of an outcome

Probability is the measure of how likely an event is. It may be described


as more likely, less likely, equally likely, or unlikely to happen. It can be
calculated based on mathematical theory (theoretical probability) or by doing
experiments and then recording the results (experimental probability).

Theoretical probability can be obtained by getting the ratio of the number


of favorable outcomes to the total number of possible outcomes. A favorable
outcome is any outcome in the event whose probability you are looking for.
Experimental probability can be obtained by getting the ratio of the number of
times an outcome is obtained to the total number of trials.

Example 1: a. If a coin is tossed 20 times, find the theoretical probability of


getting a “head”.
𝑛𝑢𝑚𝑏𝑒𝑟 𝑜𝑓 𝑓𝑎𝑣𝑜𝑟𝑎𝑏𝑙𝑒 𝑜𝑢𝑡𝑐𝑜𝑚𝑒𝑠
𝑃 (𝐸 ) = 𝑡𝑜𝑡𝑎𝑙 𝑛𝑢𝑚𝑏𝑒𝑟 𝑜𝑓 𝑝𝑜𝑠𝑠𝑖𝑏𝑙𝑒 𝑜𝑢𝑡𝑐𝑜𝑚𝑒𝑠
1
=2 (Note: E refers to the event)

193
b. If a coin is tossed 20 times and head appears 6 times, find the
experimental probability of getting a “head”.
𝑛𝑢𝑚𝑏𝑒𝑟 𝑜𝑓 𝑡𝑖𝑚𝑒𝑠 𝑡ℎ𝑒 𝑜𝑢𝑡𝑐𝑜𝑚𝑒 𝑖𝑠 𝑜𝑏𝑡𝑎𝑖𝑛𝑒𝑑
𝐸𝑥𝑝𝑒𝑟𝑖𝑚𝑒𝑛𝑡𝑎𝑙 𝑝𝑟𝑜𝑏𝑎𝑏𝑖𝑙𝑖𝑡𝑦 = 𝑡𝑜𝑡𝑎𝑙 𝑛𝑢𝑚𝑏𝑒𝑟 𝑜𝑓 𝑡𝑟𝑖𝑎𝑙𝑠
6 3
= 𝑜𝑟
20 10

Example 2: Elmo has taken the PEPT exam thrice but has never qualified
even once. Find the theoretical and experimental
probability that he will be able to qualify next time.
𝑛𝑢𝑚𝑏𝑒𝑟 𝑜𝑓 𝑓𝑎𝑣𝑜𝑟𝑎𝑏𝑙𝑒 𝑜𝑢𝑡𝑐𝑜𝑚𝑒𝑠
a. 𝑃(𝐸) = 𝑡𝑜𝑡𝑎𝑙 𝑛𝑢𝑚𝑏𝑒𝑟 𝑜𝑓 𝑝𝑜𝑠𝑠𝑖𝑏𝑙𝑒 𝑜𝑢𝑡𝑐𝑜𝑚𝑒𝑠
1
=2
𝑛𝑢𝑚𝑏𝑒𝑟 𝑜𝑓 𝑡𝑖𝑚𝑒𝑠 𝑡ℎ𝑒 𝑜𝑢𝑡𝑐𝑜𝑚𝑒 𝑖𝑠 𝑜𝑏𝑡𝑎𝑖𝑛𝑒𝑑
b. 𝐸𝑥𝑝𝑒𝑟𝑖𝑚𝑒𝑛𝑡𝑎𝑙 𝑝𝑟𝑜𝑏𝑎𝑏𝑖𝑙𝑖𝑡𝑦 = 𝑡𝑜𝑡𝑎𝑙 𝑛𝑢𝑚𝑏𝑒𝑟 𝑜𝑓 𝑡𝑟𝑖𝑎𝑙𝑠
0
= 𝑜𝑟 0
3

Try this!
You tossed 2 coins 100 times and get the following results: 2 tails =
27 outcomes, 1 head and 1 tail = 25 outcomes, 2 heads = 48
outcomes. Which of these outcomes is the theoretical probability the
same with the experimental probability?

Let’s Work Together

1. Karol picks a marble at random from a bag containing 10 red marbles, 8


blue marbles, and 12 green marbles. After he picks each marble, he
records the color and puts it back inside the bag. He repeats this
procedure 60 times and the results are as follows:

Outcomes No. of times


obtained
Green 18
Blue 16
Red 26

Find the theoretical and experimental probability of getting a blue ball.

194
2. Cassandra tossed a coin 40 times and recorded her results in a table
shown below. Find the theoretical and experimental probability of
getting a tail.
Outcomes No. of times
obtained
Head 16
Tail 24

Do on Your Own

1. Camille picks a card at random from a box containing 5 cards with the letter
“M”, 4 cards with the letter “A”, 6 cards with the letter “T”, and 2 cards
with the letter “H”. After she picks a card, she records the result and
place it back inside the box. She repeats this 20 times. Shown below is
the result of the experiment.
Outcomes M A T H
No. of times obtained 3 7 5 5

2. Ces rolled a die 100 times and recorded the results.


Outcomes 1 2 3 4 5 6
No. of times obtained 16 16 18 20 14 16
Find the theoretical and experimental probability of getting a “4”.

Let’s Analyze

1. Twelve pieces of paper with the numbers 1, 2, 2, 3, 4, 4, 5, 6, 6, 7, 8, and 8


printed on them are placed in a bag. A student chooses one without
looking. Compare the probability of choosing the number 3 with that of
choosing the number 7.
2. If a die is rolled ten times out of which thrice it gives a 5, then what will be
the experimental probability of getting a five? Compare it with
theoretical probability.

195
ACKNOWLEDGEMENT
HON. EDWIN L. OLIVAREZ
City Mayor

CONSULTANTS
MARIA MAGDALENA M. LIM, CESO V
Schools Division Superintendent

MARGARITO B.MATERUM, Ph.D.


Assistant Schools Division Superintendent

BERNARDO N. MASCARIÑA, Ed.D.


Chief - Curriculum Implementation Division

RODEL C. APOSTOL, Ed.D.


Education Program Supervisor, LRMDS

RODRIGO CONCEPCION MORALES, Ph.D


Education Program Supervisor, English

EMMA R. CUNANAN, Ed.D.


Education Program Supervisor, Mathematics

EDWIN S. DORIA, Ph.D.


Education Program Supervisor, Filipino

CORAZON A. JAVIER, Ph.D.


Education Program Supervisor, Science

GREGORIO T. CAPIRAL, Ed.D.


Education Program Supervisor, Araling Panlipunan

ANTONIO L. LAYACAN, Ed.D.


Education Program Supervisor, EPP/TLE

ARLYN M. BRIGOLA,Ph.D.
Education Program Supervisor, ESP

GLENN O. DUCTA
Education Program Supervisor, MAPEH

ANGELICA A. LIBERA, Ed.D.


Education Program Supervisor, ALS

EMERSON O. SABADLAB
Education Program Specialist for ALS

196
Illustrators

RALPH C. APOSTOL
MERVIN N. MEUDE
JHOMAR D. TAPEL
JENNIFER O. TURINGAN
ERICSON YOUNG VILLASOTO

Layout Artists

JORIZ O. MALASA
LORENA G. MANGUNE
RANILO M. PEREZ JR.
PILITA SANTOS
RHENZ NORIEL T. YEE

Proofreaders (English)

RITCHE G. BELOY
JESSIE RINA L. BERROYA
CLIFFORD N. BORDAJE
JENNIFER G. PANELO
KING VILLAUEVA

Proofreaders (Filipino)

JOCELYN D. BUENAVISTA
CRISTINA S. GALACGAC
JENNIFER A. OLAZO
SUSAN A. SEVILLA

Learning Resource Elite Team

ANGELO P. ABUGA
KATREENA A. BALUYOT
DOMINIC S. PABICO
LEONIDA L. RADEN
MA. THERESA M. RAMOS
JERLITO M. TAYLO
JONATHAN D. VECINA

197
198

You might also like